You are on page 1of 62

Exam Title : 2019 - Test 1-Polity & Curr...

Email : rahulsreedhar8787@gmail.com
Contact :

Note: If the exam is multi-lingual i.e. English and Hindi. Hindi solutions will be
after the completion of English solutions.
QUESTION 1. MTczODc4K1JhaHVsIFNyZWVkaGFyK3JhaHVsc3JlZWRoYXI4Nzg3QGdtYWlsLmNvbStRV
UVTVElPTiAw
Like India, many other countries have a written constitution. What is the basic function of a
Constitution?

a) It provides legitimate power to a President to form rules and execute them.


b) It provides a set of basic rules which provides basic rules for minimal coordination in
society.
c) It provides for a Prime Minister as a head of the government.
d) It provides law to decide punishments for crimes committed by the citizens of a country.
Correct Answer: B

Explanation

Solution (b)

The first function of a constitution is to provide a set of basic rules that allow for minimal
coordination amongst members of a society.

The second function of a constitution is to specify who has the power to make
decisions in a society. It decides how the government will be constituted.

The third function of a constitution is to set some limits on what a government can impose
on its citizens. These limits are fundamental in the sense that government may never trespass
them.

The fourth function of a constitution is to enable the government to fulfill the aspirations of a
society and create conditions for a just society.

1 st and 3 rd statements are wrong as a government can presidential or parliamentary.


Constitution codifies the authority. But what type of authority will be there depends on the kind
of polity fixed in a country.

4 th statement is incorrect because details of crimes and punishments are codified in Criminal
Procedure code and civil procedure code.

Think

· The difference between a republic and democratic

· Criminal Procedure Code

QUESTION 2. MTczODc4K1JhaHVsIFNyZWVkaGFyK3JhaHVsc3JlZWRoYXI4Nzg3QGdtYWlsLmNvbStRV
UVTVElPTiAx
Consider the following statements regarding Constituent Assembly of India:

1. Members of Constituent Assembly were directly elected by members of Provincial Legislative


Assemblies.

IASbaba
Web: http://ilp.iasbaba.com/ Score:
Email: ilp@iasbaba.com 0.00 / 150
Page 1
Exam Title : 2019 - Test 1-Polity & Curr...
Email : rahulsreedhar8787@gmail.com
Contact :

2. Representation was not provided to the Princely States of India.

3. The seats in each Province were distributed among the three main communities, Muslims,
Sikhs and general, in proportion to their respective populations.

Which of the above statements are correct?

a) 1 and 2
b) 2 and 3
c) 3 only
d) All of the above
Correct Answer: C

Explanation

Solution (c)

Formally, the Constitution was made by the Constituent Assembly which had been elected for
undivided India. It held its first sitting on 9 December1946 and reassembled as Constituent
Assembly for divided India on 14 August 1947. Its members were chosen by indirect election by
the members of the Provincial Legislative Assemblies that had been established under the
Government of India Act, 1935. The Constituent Assembly was composed roughly along the
lines suggested by the plan proposed by the committee of the British cabinet, known as the
Cabinet Mission. According to this plan:

· Each Province and each Princely State or group of States were allotted seats
proportional to their respective population roughly in the ratio of 1:10,00,000. As a result
the Provinces (that were under direct British rule) were to elect 292 members while the
Princely States were allotted a minimum of 93 seats.

· The seats in each Province were distributed among the three main communities ,
Muslims, Sikhs and general, in proportion to their respective populations.

· Members of each community in the Provincial Legislative Assembly elected their own
representatives by the method of proportional representation with single transferable
vote.

· The method of selection in the case of representatives of Princely States was to be


determined by consultation

Think

How was it ensured that each community gets representation in the Constituent Assembly of
India?

QUESTION 3. MTczODc4K1JhaHVsIFNyZWVkaGFyK3JhaHVsc3JlZWRoYXI4Nzg3QGdtYWlsLmNvbStRV
UVTVElPTiAy
After Independence India followed the idea of Universal Adult Suffrage. What is the meaning of
Universal Adult Suffrage?

a) All adults are given the right to vote without discrimination of caste, colour, race, gender or
occupation.

IASbaba
Web: http://ilp.iasbaba.com/ Score:
Email: ilp@iasbaba.com 0.00 / 150
Page 2
Exam Title : 2019 - Test 1-Polity & Curr...
Email : rahulsreedhar8787@gmail.com
Contact :

b) All adults of the country will have to suffer in the same way as the country is suffering.
c) Fundamental rights can be curtailed by the Government in case of a National Emergency.
d) Only male adult members of the country have the right to contest for elections.
Correct Answer: A

Explanation

Solution (a)

Universal Adult Suffrage

Universal Adult Suffrage or Adult franchise means that the right to vote should be given to all
adult citizens without the discrimination of caste, class, colour, religion or sex. It
demands that the right to vote should be equally available among all. To deny any class of
persons from exercising this right is to violate their right to equality.

Think

How are women voting rights in India different from voting rights of women in other countries?

QUESTION 4. MTczODc4K1JhaHVsIFNyZWVkaGFyK3JhaHVsc3JlZWRoYXI4Nzg3QGdtYWlsLmNvbStRV
UVTVElPTiAz
Which of the following countries do not have a written constitution?

1. New Zealand

2. Northern Ireland

3. Israel

4. United Kingdom

Select the code from following:

a) 1 and 4
b) 2,3 and 4
c) 1,2 and 3
d) All of the above
Correct Answer: D

Explanation

Solution (d)

Direct Question.

All these democratic countries do not have a written constitution.

For your knowledge

In most countries, ‘Constitution’ is a compact document that comprises a number of articles


about the state, specifying how the state is to be constituted and what norms it should follow. W

IASbaba
Web: http://ilp.iasbaba.com/ Score:
Email: ilp@iasbaba.com 0.00 / 150
Page 3
Exam Title : 2019 - Test 1-Polity & Curr...
Email : rahulsreedhar8787@gmail.com
Contact :

hen we ask for the constitution of a country we are usually referring to this document. But some
countries, the United Kingdom for instance, do not have one single document that can be called
the Constitution. Rather they have a series of documents and decisions that, taken collectively,
are referred to as the constitution.

QUESTION 5. MTczODc4K1JhaHVsIFNyZWVkaGFyK3JhaHVsc3JlZWRoYXI4Nzg3QGdtYWlsLmNvbStRV
UVTVElPTiA0
The summary of the principles on which the Nationalist movement was based, was brought to
the Constituent Assembly through the Objectives Resolution. The objectives Resolution in the
Assembly was moved by

a) Motilal Nehru
b) M N Roy
c) Sardar Vallabh Bhai Patel
d) Jawaharlal Nehru
Correct Answer: D

Explanation

Solution (d)

Objectives Resolution

Objective resolution was moved by Jawaharlal Nehru in 1946 it laid down the basic principles
and the ideas on which the Indian constitution has to be made by the assembly. It was the
objectives resolution that gave institutional expression to the fundamental commitments that is
equality sovereignty and liberty.

Think

Preamble of Indian Constitution

QUESTION 6. MTczODc4K1JhaHVsIFNyZWVkaGFyK3JhaHVsc3JlZWRoYXI4Nzg3QGdtYWlsLmNvbStRV
UVTVElPTiA1
Which of the following personalities were not the part of Constituent Assembly of India?

a) Mahatma Gandhi
b) Dr B R Ambedkar
c) Dr Rajendra Prasad
d) Maulana Hasrat Mohani
Correct Answer: A

Explanation

Solution (a)

Most of the prominent leaders were part of the Constituent Assembly of India when it was
formed except two tall leaders – Mahatma Gandhi and Mohammad Ali Jinnah.

IASbaba
Web: http://ilp.iasbaba.com/ Score:
Email: ilp@iasbaba.com 0.00 / 150
Page 4
Exam Title : 2019 - Test 1-Polity & Curr...
Email : rahulsreedhar8787@gmail.com
Contact :

Think

Being the most prominent leader of his time, why didn’t Mahatma Gandhi became part of
Constituent Assembly of India?

QUESTION 7. MTczODc4K1JhaHVsIFNyZWVkaGFyK3JhaHVsc3JlZWRoYXI4Nzg3QGdtYWlsLmNvbStRV
UVTVElPTiA2
Which of the following provisions in Indian Constitution were adopted from Irish Constitution?

a) First Past the post system


b) Fundamental Rights
c) Idea of Residual Power
d) Directive Principles of State Policy
Correct Answer: D
Explanation

Solution (d)

IASbaba
Web: http://ilp.iasbaba.com/ Score:
Email: ilp@iasbaba.com 0.00 / 150
Page 5
Exam Title : 2019 - Test 1-Polity & Curr...
Email : rahulsreedhar8787@gmail.com
Contact :

Think

How were different provisions adopted from the different constitutions of the World, indianised
to make them fit into Indian scenario?

IASbaba
Web: http://ilp.iasbaba.com/ Score:
Email: ilp@iasbaba.com 0.00 / 150
Page 6
Exam Title : 2019 - Test 1-Polity & Curr...
Email : rahulsreedhar8787@gmail.com
Contact :

QUESTION 8. MTczODc4K1JhaHVsIFNyZWVkaGFyK3JhaHVsc3JlZWRoYXI4Nzg3QGdtYWlsLmNvbStRV
UVTVElPTiA3
Which of the following statements are correct regarding the Fundamental Rights present in
Indian Constitution?

1. They prevent the Government to encroach over the rights of Individuals

2. They prevent the citizens to encroach over the Rights of Government

Select the code from following:

a) 1 only
b) 2 only
c) Both 1 and 2
d) Neither 1 nor 2
Correct Answer: A

Explanation

Solution (a)

A democracy must ensure that individuals have certain rights and that the government will
always recognise these rights. Therefore it is often a practice in most democratic countries to
list the rights of the citizens in the constitution itself. Such a list of rights mentioned and
protected by the constitution is called the ‘bill of rights’. A bill of rights prohibits government
from thus acting against the rights of the individuals and ensures a remedy in case there is
violation of these rights.

Think

Were Fundamental Duties added in the Constitution to balance the Fundamental Rights?

QUESTION 9. MTczODc4K1JhaHVsIFNyZWVkaGFyK3JhaHVsc3JlZWRoYXI4Nzg3QGdtYWlsLmNvbStRV
UVTVElPTiA4
Which of the following made the demand for Fundamental Rights for the first time?

a) Jinnah’s 14 point report


b) Nehru Report in 1928
c) Congress resolution in 1885
d) Congress Resolution in 1931
Correct Answer: B

Explanation

Solution (b)

During our freedom struggle, the leaders of the freedom movement had realized the importance
of rights and demanded that the British rulers should respect rights of the people. The Motilal
Nehru committee had demanded a bill of rights as far back as in 1928. It was therefore, natural
that when India became independent and the Constitution was being prepared, there were no

IASbaba
Web: http://ilp.iasbaba.com/ Score:
Email: ilp@iasbaba.com 0.00 / 150
Page 7
Exam Title : 2019 - Test 1-Polity & Curr...
Email : rahulsreedhar8787@gmail.com
Contact :

two opinions on the inclusion and protection of rights in the Constitution. The Constitution
listed the rights that would be specially protected and called them ‘fundamental rights’.

Think

· Nehru Report 1928

· Karachi session of Congress

QUESTION 10. MTczODc4K1JhaHVsIFNyZWVkaGFyK3JhaHVsc3JlZWRoYXI4Nzg3QGdtYWlsLmNvbStRV


UVTVElPTiA5
Which of the following are correct differences between Fundamental Rights and Other Rights
available to us?

1. While ordinary legal rights are protected and enforced by ordinary law, Fundamental
Rights are protected and guaranteed by the constitution

2. Ordinary rights may be changed by the legislature by ordinary process of law


making, but a fundamental right may only be changed by amending the Constitution

Select the code from following:

a) 1 only
b) 2 only
c) Both 1 and 2
d) Neither 1 nor 2
Correct Answer: C

Explanation

Solution (c)

Fundamental Rights are different from other rights available to us. While ordinary legal rights
are protected and enforced by ordinary law, Fundamental Rights are protected and
guaranteed by the constitution of the country. Ordinary rights may be changed by the
legislature by ordinary process of law making, but a fundamental right may only be
changed by amending the Constitution itself. Besides this, no organ of the government can act
in a manner that violates them.

Think

Why some Rights were made Fundamental in Constitution while others were not?

QUESTION 11. MTczODc4K1JhaHVsIFNyZWVkaGFyK3JhaHVsc3JlZWRoYXI4Nzg3QGdtYWlsLmNvbStRVUVTV


ElPTiAxMA==
Which of the Fundamental Rights under Right to Equality provide Constitutional Provision for
Reservation for depressed classes?

a) Article 17

IASbaba
Web: http://ilp.iasbaba.com/ Score:
Email: ilp@iasbaba.com 0.00 / 150
Page 8
Exam Title : 2019 - Test 1-Polity & Curr...
Email : rahulsreedhar8787@gmail.com
Contact :

b) Article 15
c) Article 16
d) Article 19
Correct Answer: C

Explanation

Solution (c)

Equality of opportunity means that all sections of the society enjoy equal opportunities. But in a
society where there are various kinds of social inequalities.

The Constitution clarifies that the government can implement special schemes and measures fo
r improving the conditions of certain sections of society: children, women, and the socially and
educationally backward classes.

Article 16(4) of the constitution explicitly clarifies that a policy like reservation will
not be seen as a violation of right to equality . If you see the spirit of the Constitution, this
is required for the fulfilment of the right to equality of opportunity.

Think

Do you think there needs to be a change in the process of Reservation?

QUESTION 12. MTczODc4K1JhaHVsIFNyZWVkaGFyK3JhaHVsc3JlZWRoYXI4Nzg3QGdtYWlsLmNvbStRVUVTV


lPTiAxMQ==
Which of the following Rights have been provided by the Constitution to ensure fair trials in
court?

1. No person would be punished for the same offence more than once

2. No law shall declare any action as illegal from a backdate

3. No person shall be asked to give evidence against himself or herself.

Select the code from following:

a) 1 and 2
b) 2 and 3
c) 1 and 3
d) All of the above
Correct Answer: D

Explanation

Solution (d)

To ensure a fair trial in courts, the Constitution has provided three rights:

· No person would be punished for the same offence more than once,

IASbaba
Web: http://ilp.iasbaba.com/ Score:
Email: ilp@iasbaba.com 0.00 / 150
Page 9
Exam Title : 2019 - Test 1-Polity & Curr...
Email : rahulsreedhar8787@gmail.com
Contact :

· No law shall declare any action as illegal from a backdate, and

· No person shall be asked to give evidence against himself or herself.

Think

Preventive Detention

QUESTION 13. MTczODc4K1JhaHVsIFNyZWVkaGFyK3JhaHVsc3JlZWRoYXI4Nzg3QGdtYWlsLmNvbStRVUVTV


ElPTiAxMg==
Which of the following statements are correct regarding Indian law

a) An accused is not considered guilty till the court has found he is guilty
b) An accused is considered guilty till he proves in court that he is not guilty
c) An accused can be indefinitely kept for interrogation by police without presented before a
magistrate.
d) None of the above
Correct Answer: A

Explanation

Solution (a)

Our Constitution ensures that persons accused of various offences would also get sufficient
protection. We often tend to believe that anyone who is charged with some offence is guilty.
However, no one is guilty unless the court has found that person guilty of an offence. It is also
necessary that a person accused of any crime should get adequate opportunity to defend herself
or himself.

Think

Do you think many criminals get away because of the above law?

QUESTION 14. MTczODc4K1JhaHVsIFNyZWVkaGFyK3JhaHVsc3JlZWRoYXI4Nzg3QGdtYWlsLmNvbStRVUVTV


lPTiAxMw==
Which of the following Fundamental Rights have made selling and buying of human beings
unconstitutional?

a) Article 21
b) Article 23
c) Article 24
d) Article 32
Correct Answer: B
Explanation

Solution (b)

IASbaba
Web: http://ilp.iasbaba.com/ Score:
Email: ilp@iasbaba.com 0.00 / 150
Page 10
Exam Title : 2019 - Test 1-Polity & Curr...
Email : rahulsreedhar8787@gmail.com
Contact :

Article 23 - Prohibition of traffic in human beings and forced labour

· Traffic in human beings and begar and other similar forms of forced labour are prohibited and
any contravention of this provision shall be an offence punishable in accordance with law

· Nothing in this article shall prevent the State from imposing compulsory service for public
purpose, and in imposing such service the State shall not make any discrimination on grounds
only of religion, race, caste or class or any of them

QUESTION 15. MTczODc4K1JhaHVsIFNyZWVkaGFyK3JhaHVsc3JlZWRoYXI4Nzg3QGdtYWlsLmNvbStRVUVTV


ElPTiAxNA==
Under Fundamental Rights, Indian Constitution has provided Freedom of Faith and Worship.
Which of the following statements are correct regarding this?

1. A person may choose any religion or may choose not to follow any religion.

2. It provides the freedom to profess, follow and propagate any religion.

3. It inhibits the government to interfere in any religious matter.

Select the code from below:

a) 1 and 2
b) 2 and 3
c) 1 and 3
d) All of the above
Correct Answer: A

Explanation

Solution (a)

Freedom of faith and worship

In India, everyone is free to choose a religion and practice that religion. Freedom of religion
also includes the freedom of conscience. This means that a person may choose any religion or
may choose not to follow any religion. Freedom of religion includes the freedom to profess,
follow and propagate any religion. Freedom of religion is subject to certain limitations. The
government can impose restrictions on the practice of freedom of religion in order to protect
public order, morality and health. This means that the freedom of religion is not an unlimited
right. The government can interfere in religious matters for rooting out certain social evils. For
example in the past, the government has taken steps banning practices like sati, bigamy or
human sacrifice. Such restrictions cannot be opposed in the name of interference in right to
freedom of religion.

Think

How is India’s secularism different from western idea of secularism?

IASbaba
Web: http://ilp.iasbaba.com/ Score:
Email: ilp@iasbaba.com 0.00 / 150
Page 11
Exam Title : 2019 - Test 1-Polity & Curr...
Email : rahulsreedhar8787@gmail.com
Contact :

QUESTION 16. MTczODc4K1JhaHVsIFNyZWVkaGFyK3JhaHVsc3JlZWRoYXI4Nzg3QGdtYWlsLmNvbStRVUVTV


ElPTiAxNQ==
Consider the following statements:

1. In a direct democracy people of a country elect their representatives to legislate.

2. India is a direct democracy

Which of the above statement(s) is/are correct?

a) 1 only
b) 2 only
c) Both 1 and 2
d) Neither 1 nor 2
Correct Answer: D

Explanation

Solution (d)

We often distinguish between direct and indirect democracy. A direct democracy is one where
the citizens directly participate in the day-to-day decision - making and in the running of the
government.

India is an in – direct democracy where people elect their representatives which in turn make
the laws for the country.

Think

Can you think of some countries where direct democracy is practiced?

QUESTION 17. MTczODc4K1JhaHVsIFNyZWVkaGFyK3JhaHVsc3JlZWRoYXI4Nzg3QGdtYWlsLmNvbStRVUVTV


ElPTiAxNg==
After Independence, India adopted the first past the post system of elections. Which of the
following statements are correct regarding FPTP system?

1. Voters vote for a candidate rather than a party.

2. Candidate who wins may not get the majority votes (50% + 1).

3. More than one representative may be elected from one constituency.

4. Every party gets a seat in legislature in proportion to the percentage of votes that it has got.

Select the code from following:

a) 1,2 and 3
b) 2,3 and 4
c) 1,3 and 4
d) 1 and 2
Correct Answer: D

IASbaba
Web: http://ilp.iasbaba.com/ Score:
Email: ilp@iasbaba.com 0.00 / 150
Page 12
Exam Title : 2019 - Test 1-Polity & Curr...
Email : rahulsreedhar8787@gmail.com
Contact :

Explanation

Solution (d)

Think

IASbaba
Web: http://ilp.iasbaba.com/ Score:
Email: ilp@iasbaba.com 0.00 / 150
Page 13
Exam Title : 2019 - Test 1-Polity & Curr...
Email : rahulsreedhar8787@gmail.com
Contact :

Why did India chose to adopt the First Past the pole system?

QUESTION 18. MTczODc4K1JhaHVsIFNyZWVkaGFyK3JhaHVsc3JlZWRoYXI4Nzg3QGdtYWlsLmNvbStRVUVTV


ElPTiAxNw==
Who of the following was the first Chief election Commissioner of India?

a) G V Mavalnkar
b) Sukumar Sen
c) V P Menon
d) H J Kania
Correct Answer: B

Explanation

Solution (b)

Shri Sukumar Sen was the first Chief Election Commissioner of the Election Commission of
India.

He successfully finished the task of conducting the first General elections after independence.

Think

What were the difficulties faced by the first election commission of India while conducting the
first General Elections?

QUESTION 19. MTczODc4K1JhaHVsIFNyZWVkaGFyK3JhaHVsc3JlZWRoYXI4Nzg3QGdtYWlsLmNvbStRVUVTV


ElPTiAxOA==
Which of the following bodies decide that which of the constituencies in India will be reserved
constituencies?

a) NITI aayog
b) Election Commission
c) Delimitation Commission
d) National Commission of SC and ST
Correct Answer: C

Explanation

Solution (c)

Delimitation Commission

The Delimitation Commission is appointed by the President of India and works in collaboration
with the Election Commission of India. It is appointed for the purpose of drawing up the
boundaries of constituencies all over the country. A quota of constituencies to be reserved in
each State is fixed depending on the proportion of SC or ST in that State. After drawing the

IASbaba
Web: http://ilp.iasbaba.com/ Score:
Email: ilp@iasbaba.com 0.00 / 150
Page 14
Exam Title : 2019 - Test 1-Polity & Curr...
Email : rahulsreedhar8787@gmail.com
Contact :

boundaries, the Delimitation Commission looks at the composition of population in each


constituency. Those constituencies that have the highest proportion of Scheduled Tribe
population are reserved for ST.

In the case of Scheduled Castes, the Delimitation Commission looks at two things. It picks
constituencies that have higher proportion of Scheduled Caste population. But it also spreads
these constituencies in different regions of the State. This is done because the Scheduled Caste
population is generally spread evenly throughout the country. These reserved constituencies
can be rotated each time the Delimitation exercise is undertaken.

Think

Why did India trashed the concept of separate electorate after independence?

QUESTION 20. MTczODc4K1JhaHVsIFNyZWVkaGFyK3JhaHVsc3JlZWRoYXI4Nzg3QGdtYWlsLmNvbStRVUVTV


ElPTiAxOQ==
Before the First legislature was elected in India by General elections of 1951 – 52, which body
acted as the legislature for India?

a) Legislature of Britain
b) Supreme Court of India
c) Executive council of Viceroy
d) Constituent Assembly
Correct Answer: D

Explanation

Solution (d)

Before the election of the first legislature, The Constituent Assembly, apart from the function of
drafting the Constitution of India, also acted as the interim legislature for making general laws
for day to day working of the interim government.

QUESTION 21. MTczODc4K1JhaHVsIFNyZWVkaGFyK3JhaHVsc3JlZWRoYXI4Nzg3QGdtYWlsLmNvbStRVUVTV


ElPTiAyMA==
Consider the following statements regarding the citizenship in India.

1. The constitution of India provides for single citizenship to all and no amendments regarding
this can be made whatsoever.

2. Amendments related to citizenship in India can be made only after consulting half of the
states.

3. In India no discrimination is allowed in matters related to awarding citizenship.

Choose the correct answer

a) 1 and 2 only
b) 2 and 3 only

IASbaba
Web: http://ilp.iasbaba.com/ Score:
Email: ilp@iasbaba.com 0.00 / 150
Page 15
Exam Title : 2019 - Test 1-Polity & Curr...
Email : rahulsreedhar8787@gmail.com
Contact :

c) 2 only
d) None of the above
Correct Answer: D

Explanation

solution (d)

The Indian Constitution is federal and envisages a dual polity (Centre and states), it provides
for only a single citizenship, that is, the Indian citizenship.

Do you know?

In countries like USA, each person is not only a citizen of USA but also a citizen of the
particular state to which he belongs. Thus, he owes allegiance to both and enjoys dual sets of
rights—one conferred by the National government and another by the state government. Any
amendments related to citizenship can be made through simple majority of the
parliament .

In India, all citizens irrespective of the state in which they are born or reside enjoy the same
political and civil rights of citizenship all over the country and no discrimination is made
between them excepting in few cases like tribal areas, Jammu and Kashmir, and so on.

QUESTION 22. MTczODc4K1JhaHVsIFNyZWVkaGFyK3JhaHVsc3JlZWRoYXI4Nzg3QGdtYWlsLmNvbStRVUVTV


ElPTiAyMQ==
The idea of single citizenship is borrowed from which of the following?

a) American constitution
b) British constitution
c) Government of India act 1935
d) It is an Indian innovation
Correct Answer: B
Explanation

solution (b)

Source Features borrowed

Fundamental rights, Independence of


judiciary, Judicial review, Impeachment of
American constitution
president, Removal of supreme court and
high court judges, post of vice president

IASbaba
Web: http://ilp.iasbaba.com/ Score:
Email: ilp@iasbaba.com 0.00 / 150
Page 16
Exam Title : 2019 - Test 1-Polity & Curr...
Email : rahulsreedhar8787@gmail.com
Contact :

Parliamentary government, rule of law,


legislative procedure, single citizenship,
British constitution
cabinet system, prerogative writs,
parliamentary privileges and bicameralism

Federal scheme, office of governor,


judiciary, public service commission,
Government of India act 1935
emergency provisions and administrative
details

QUESTION 23. MTczODc4K1JhaHVsIFNyZWVkaGFyK3JhaHVsc3JlZWRoYXI4Nzg3QGdtYWlsLmNvbStRVUVTV


ElPTiAyMg==
Which of the following rights are enjoyed only by the citizens of India?

1. Right to vote in elections to the Lok Sabha and state legislative assembly.

2. Right to hold the office of president of India.

3. Right to contest for the membership of the Parliament and the State legislature.

4. Right to protection against arrest and detention.

Choose the correct code

a) 1 and 3 only
b) 1,2 and 3 only
c) 1,2 and 4 only
d) 2,3 and 4 only
Correct Answer: B

Explanation

solution (b)

Citizens are full members of the Indian State and owe allegiance to it. They enjoy all civil and
political rights. Aliens, on the other hand, are the citizens of some other state and hence, do not
enjoy all the civil and political rights.

Do you know?

Aliens are of two categories— friendly aliens or enemy aliens . Friendly aliens are the
subjects of those countries that have cordial relations with India.

The Constitution confers the following rights and privileges on the citizens of India (and
denies the same to aliens):

1. Right against discrimination on grounds of religion, race, caste, sex or place of birth (Article
15).

2. Right to equality of opportunity in the matter of public employment (Article 16).

IASbaba
Web: http://ilp.iasbaba.com/ Score:
Email: ilp@iasbaba.com 0.00 / 150
Page 17
Exam Title : 2019 - Test 1-Polity & Curr...
Email : rahulsreedhar8787@gmail.com
Contact :

3. Right to freedom of speech and expression, assembly, association, movement, residence and
profession (Article 19).

4. Cultural and educational rights (Articles 29 and 30).

5. Right to vote in elections to the Lok Sabha and state legislative assembly.

6. Right to contest for the membership of the Parliament and the state legislature.

7. Eligibility to hold certain public offices, that is, President of India, Vice-President of India,
judges of the Supreme Court and the high courts, governor of states, attorney general of India
and advocate general of states.

Note:

Both citizens and aliens enjoy protection against arrest and detention (article 22)

QUESTION 24. MTczODc4K1JhaHVsIFNyZWVkaGFyK3JhaHVsc3JlZWRoYXI4Nzg3QGdtYWlsLmNvbStRVUVTV


ElPTiAyMw==
Consider the following regarding the recently proposed citizenship amendment bill 2016.

1. The Bill amends the Citizenship Act, 1955 to make all illegal migrants from Afghanistan,
Bangladesh and Pakistan, eligible for citizenship.

2. The Bill relaxes the requirement of citizenship by naturalisation from eleven years to six
years for persons belonging to the above mentioned three countries.

3. The bill aims to merge the Persons of Indian Origin card and Overseas Citizen of India card
schemes into a single scheme.

Choose the correct code.

a) 1 and 2 only
b) 1 and 3 only
c) 2 only
d) 2 and 3 only
Correct Answer: C
Explanation

solution (c)

Highlights of the Bill

1) The Bill amends the Citizenship Act, 1955 to make illegal migrants who are Hindus, Sikhs,
Buddhists, Jains, Parsis and Christians from Afghanistan, Bangladesh and Pakistan, eligible for
citizenship.

2) Under the Act, one of the requirements for citizenship by naturalisation is that the applicant
must have resided in India during the last 12 months, and for 11 of the previous 14 years. The
Bill relaxes this 11 year requirement to six years for persons belonging to the same six religions
and three countries.

IASbaba
Web: http://ilp.iasbaba.com/ Score:
Email: ilp@iasbaba.com 0.00 / 150
Page 18
Exam Title : 2019 - Test 1-Polity & Curr...
Email : rahulsreedhar8787@gmail.com
Contact :

3) The Bill provides that the registration of Overseas Citizen of India (OCI) cardholders may be
cancelled if they violate any law

Do you know?

In 2011, the Government of India announced its decision to merge the PIO card and OCI
card schemes into a single scheme. This new scheme is proposed to be called as the
Overseas Indian Card Holders Scheme. In this regard, the Citizenship (Amendment) Bill,
2011 , is under consideration of the Parliament.

QUESTION 25. MTczODc4K1JhaHVsIFNyZWVkaGFyK3JhaHVsc3JlZWRoYXI4Nzg3QGdtYWlsLmNvbStRVUVTV


ElPTiAyNA==
Which of the following are the grounds mentioned in the constitution for acquiring the
Citizenship?

1. Citizenship by birth

2. Citizenship by descent

3. Citizenship by naturalisation

4. Citizenship by acquisition of territory

Choose the correct code from below

a) 1,2 and 3 only


b) 1,2 and 4 only
c) All of the above
d) None of the above
Correct Answer: D

Explanation

solution (d)

The Constitution deals with the citizenship from Articles 5 to 11 under Part II . However, it
contains neither any permanent nor any elaborate provisions in this regard. It only identifies
the persons who became citizens of India at its commencement (i.e., on January 26, 1950). It
does not deal with the problem of acquisition or loss of citizenship subsequent to its
commencement.

Do you know?

The Citizenship Act of 1955 prescribes five ways of acquiring citizenship, namely, birth,
descent, registration, naturalisation and incorporation of territory.

QUESTION 26. MTczODc4K1JhaHVsIFNyZWVkaGFyK3JhaHVsc3JlZWRoYXI4Nzg3QGdtYWlsLmNvbStRVUVTV


ElPTiAyNQ==

IASbaba
Web: http://ilp.iasbaba.com/ Score:
Email: ilp@iasbaba.com 0.00 / 150
Page 19
Exam Title : 2019 - Test 1-Polity & Curr...
Email : rahulsreedhar8787@gmail.com
Contact :

Who of the following is the competent authority in India for awarding citizenship by naturalisati
on ?

a) President
b) Foreign Ministry
c) Home ministry
d) Parliament
Correct Answer: C

Explanation

solution (c)

The Citizenship Act of 1955 prescribes five ways of acquiring citizenship, namely birth, descent,
registration, naturalisation and incorporation of territory

The Central Government (Ministry of Home Affairs) may, on an application , grant a


certificate of naturalisation to any person (not being an illegal migrant) if he possesses the
following qualifications:

(a) That he is not a subject or citizen of any country where citizens of India are prevented from
becoming subjects or citizens of that country by naturalisation .

(b) that, if he is a citizen of any country, he undertakes to renounce the citizenship of that
country in the event of his application for Indian citizenship being accepted.

(c) that he has either resided in India or been in the service of a Government in India or partly
the one and partly the other, throughout the period of twelve months immediately preceding the
date of the application.

(d) that during the fourteen years immediately preceding the said period of twelve months, he
has either resided in India or been in the service of a Government in India, or partly the one and
partly the other, for periods amounting in the aggregate to not less than eleven years.

(e) That he is of good character

(f) That he has an adequate knowledge of a language specified in the Eighth Schedule to the
Constitution and

(g) that in the event of a certificate of naturalisation being granted to him, he intends to reside
in India, or to enter into or continue in, service under a Government in India or under an
international organisation of which India is a member or under a society, company or body of
persons established in India.

However, the Government of India may waive all or any of the above conditions for natur
alisation in the case of a person who has rendered distinguished service to the science,
philosophy, art, literature, world peace or human progress.

Every naturalised citizen must take an oath of allegiance to the Constitution of India.

QUESTION 27. MTczODc4K1JhaHVsIFNyZWVkaGFyK3JhaHVsc3JlZWRoYXI4Nzg3QGdtYWlsLmNvbStRVUVTV


ElPTiAyNg==

IASbaba
Web: http://ilp.iasbaba.com/ Score:
Email: ilp@iasbaba.com 0.00 / 150
Page 20
Exam Title : 2019 - Test 1-Polity & Curr...
Email : rahulsreedhar8787@gmail.com
Contact :

By virtue of being citizens of India, the people of India enjoy which of the following rights?

1. Political rights only

2. Civil rights only

Choose the correct code

a) 1 only
b) 2 only
c) Both 1 and 2
d) Neither 1 nor 2
Correct Answer: C

Explanation

solution (c)

In India, all citizens irrespective of the state in which they are born or reside enjoy the same po
litical and civil rights of citizenship all over the country and no discrimination is made
between them. However, this general rule of absence of discrimination is subject to some
exceptions, namely

1. The Parliament (under Article 16) can prescribe residence within a state or union territory as
a condition for certain employments or appointments in that state or union territory, or local
authority or other authority within that state or union territory. Accordingly, the Parliament
enacted the Public Employment (Requirement as to Residence) Act, 1957 and thereby authorise
d the Government of India to prescribe residential qualification only for appointment to non- Ga
zetted posts in Andhra Pradesh, Himachal Pradesh, Manipur and Tripura. As this Act expired in
1974, there is no such provision for any state except Andhra Pradesh.

2. The Constitution (under Article 15) prohibits discrimination against any citizen on grounds of
religion, race, caste, sex or place of birth and not on the ground of residence. This means that
the state can provide special benefits or give preference to its residents in matters that do not
come within the purview of the rights given by the Constitution to the Indian citizens. For
example, a state may offer concession in fees for education to its residents.

3. The freedom of movement and residence (under Article 19) is subjected to the protection of
interests of any schedule tribe. In other words, the right of outsiders to enter, reside and settle
in tribal areas is restricted. Of course, this is done to protect the distinctive culture, language,
customs and manners of schedule tribes and to safeguard their traditional vocation and
property against exploitation.

4. In the case of Jammu and Kashmir, the state legislature is empowered to define the persons
who are permanent residents of the state and confer any special rights and privileges in
matters of employment under the state government, acquisition of immovable property in the
state, settlement in the state and scholarships and such other forms of aid provided by the state
government.

QUESTION 28. MTczODc4K1JhaHVsIFNyZWVkaGFyK3JhaHVsc3JlZWRoYXI4Nzg3QGdtYWlsLmNvbStRVUVTV


ElPTiAyNw==
During which of the following instances, a person loses his citizenship by termination?

IASbaba
Web: http://ilp.iasbaba.com/ Score:
Email: ilp@iasbaba.com 0.00 / 150
Page 21
Exam Title : 2019 - Test 1-Polity & Curr...
Email : rahulsreedhar8787@gmail.com
Contact :

a) When he make a declaration renouncing his Indian citizenship to the central government.
b) When he voluntarily acquires the citizenship of another country.
c) When he has obtained the citizenship by fraud.
d) When he has shown disloyalty to the Constitution of India.
Correct Answer: B

Explanation

solution (b)

The Citizenship Act, 1955 , prescribes three ways of losing citizenship whether acquired
under the Act or prior to it under the Constitution, namely, renunciation, termination and
deprivation:

1. By Renunciation - Any citizen of India of full age and capacity can make a declaration
renouncing his Indian citizenship. Upon the registration of that declaration, that person ceases
to be a citizen of India. However, if such a declaration is made during a war in which India is
engaged, its registration shall be withheld by the Central Government. Further, when a person
renounces his Indian citizenship, every minor child of that person also loses Indian citizenship.
However, when such a child attains the age of eighteen, he may resume Indian citizenship.

2. By Termination - When an Indian citizen voluntarily (consciously, knowingly and without


duress, undue influence or compulsion) acquires the citizenship of another country, his Indian
citizenship automatically terminates. This provision, however, does not apply during a war in
which India is engaged.

3. By Deprivation - It is a compulsory termination of Indian citizenship by the Central


government, If

(a) the citizen has obtained the citizenship by fraud

(b) the citizen has shown disloyalty to the Constitution of India

(c) the citizen has unlawfully traded or communicated with the enemy during a war

(d) the citizen has, within five years after registration or naturalisation , been imprisoned in any
country for two years and

(e) the citizen has been ordinarily resident out of India for seven years continuously.

QUESTION 29. MTczODc4K1JhaHVsIFNyZWVkaGFyK3JhaHVsc3JlZWRoYXI4Nzg3QGdtYWlsLmNvbStRVUVTV


ElPTiAyOA==
Consider the following statements regarding the reorganisation of Indian states.

1. Andhra Pradesh became the first state to be reorganised on the linguistic basis following the
recommendation of Fazl Ali commission.

2. Dhar commission gave in to the idea of reorganisation of states on linguistic basis where as
JVP committee opposed this idea.

3. The idea of Linguistic and cultural homogeneity as the basis for reorganisation was the
outcome of Fazl Ali commission.

IASbaba
Web: http://ilp.iasbaba.com/ Score:
Email: ilp@iasbaba.com 0.00 / 150
Page 22
Exam Title : 2019 - Test 1-Polity & Curr...
Email : rahulsreedhar8787@gmail.com
Contact :

Choose the correct code.

a) 1 and 2 only
b) 2 and 3 only
c) 3 only
d) None of the above
Correct Answer: C

Explanation

solution (c)

There has been a demand from different regions, particularly South India, for reorganisation of
states on linguistic basis. Accordingly, in June 1948, the Government of India appointed the Lin
guistic Provinces Commission under the chairmanship of S K Dhar to examine the
feasibility of this. The commission submitted its report in December 1948 and recommended
the reorganisation of states on the basis of administrative convenience rather than
linguistic factor.

This created much resentment and led to the appointment of another Linguistic Provinces
Committee by the Congress in December 1948 itself to examine the whole question afresh. It
consisted of Jawaharlal Nehru, Vallahbhai Patel and Pattabhi Sitaramayya and hence,
was popularly known as JVP Committee .

It submitted its report in April 1949 and formally rejected language as the basis for reorgan
isation of states. However, in October 1953, the Government of India was forced to create the
first linguistic state, known as Andhra state, by separating the Telugu speaking areas from the
Madras state.

The creation of Andhra state intensified the demand from other regions for creation of states on
linguistic basis. This forced the Government of India to appoint (in December 1953) a three-
member States Reorganisation Commission under the chairmanship of Fazl Ali to re-examine
the whole question.

It submitted its report in September 1955 and broadly accepted language as the basis of reo
rganisation of states . But, it rejected the theory of ‘one language–one state’.

It is clear that Andhra Pradesh was reorganised before the commencement of Fazl Ali
commission.

It identified four major factors that can be taken into account in any scheme of reorganisation o
f states:

(a) Preservation and strengthening of the unity and security of the country.

(b) Linguistic and cultural homogeneity.

(c) Financial, economic and administrative considerations.

(d) Planning and promotion of the welfare of the people in each state as well as of the nation as
a whole.

Do you know?

IASbaba
Web: http://ilp.iasbaba.com/ Score:
Email: ilp@iasbaba.com 0.00 / 150
Page 23
Exam Title : 2019 - Test 1-Polity & Curr...
Email : rahulsreedhar8787@gmail.com
Contact :

The Indian Independence Act (1947) created two independent and separate dominions of India
and Pakistan and gave three options to the princely states viz., joining India, joining Pakistan or
remaining independent. Of the 552 princely states situated within the geographical boundaries
of India, 549 joined India and the remaining (Hyderabad, Junagarh and Kashmir) refused to join
India. However, in course of time, they were also integrated with India—Hyderabad by means of
police action, Junagarh by means of referendum and Kashmir by the Instrument of Accession.

QUESTION 30. MTczODc4K1JhaHVsIFNyZWVkaGFyK3JhaHVsc3JlZWRoYXI4Nzg3QGdtYWlsLmNvbStRVUVTV


ElPTiAyOQ==
Suppose the parliament wants to create a new state by separating a territory from a particular
state , then which of the following becomes compulsory provision in going ahead with the
procedure?

1. A bill contemplating the formation of new state can be introduced in the Parliament only with
the prior recommendation of the President.

2. President has to refer the bill for the creation of new state to the state legislature concerned
for expressing its views within a specified period.

3. Parliament has to take the permission of the state concerned before framing a bill on the
creation of new state.

Choose the correct code

a) 1 only
b) 2 only
c) 1 and 2 only
d) 1 and 3 only
Correct Answer: C

Explanation

solution (c)

Article 3 of the constitution of India authorises the Parliament to:

1) Form a new state by separation of territory from any state or by uniting two or more states or
parts of states or by uniting any territory to a part of any state.

2) Increase the area of any state,

3) Diminish the area of any state,

4) Alter the boundaries of any state,

5) Alter the name of any state.

Do you know?

Article 3 lays down two conditions in the above case

IASbaba
Web: http://ilp.iasbaba.com/ Score:
Email: ilp@iasbaba.com 0.00 / 150
Page 24
Exam Title : 2019 - Test 1-Polity & Curr...
Email : rahulsreedhar8787@gmail.com
Contact :

1) A bill contemplating the above changes can be introduced in the Parliament only with the
prior recommendation of the President .

2) Before recommending the bill, the President has to refer the same to the state
legislature concerned for expressing its views within a specified period.

Further, the power of Parliament to form new states includes the power to form a new state or
union territory by uniting a part of any state or union territory to any other state or union
territory.

The President (or Parliament) is not bound by the views of the state legislature and
may either accept or reject them, even if the views are received in time. Further, it is not
necessary to make a fresh reference to the state legislature every time an amendment to the
bill is moved and accepted in Parliament.

In case of a union territory, no reference need be made to the concerned legislature to


ascertain its views and the Parliament can itself take any action as it deems fit. It is thus clear
that the Constitution authorises the Parliament to form new states or alter the areas,
boundaries or names of the existing states without their consent. In other words, the Parliame
nt can redraw the political map of India according to its will .

Hence, the territorial integrity or continued existence of any state is not guaranteed by
the Constitution . Therefore, India is rightly described as ‘an indestructible union of
destructible states’. The Union government can destroy the states whereas the state
governments cannot destroy the Union.

In USA, on the other hand, the territorial integrity or continued existence of a state is
guaranteed by the Constitution. The American Federal government cannot form new states or
alter the borders of existing states without the consent of the states concerned. That is why the
USA is described as ‘an indestructible union of indestructible states.’

QUESTION 31. MTczODc4K1JhaHVsIFNyZWVkaGFyK3JhaHVsc3JlZWRoYXI4Nzg3QGdtYWlsLmNvbStRVUVTV


ElPTiAzMA==
Who of the following is the Head of the Government in India?

a) President
b) Prime Minister
c) Speaker of Lok Sabha
d) Chief Justice of India
Correct Answer: B

Explanation

Solution (b)

The Prime Minister of India, as addressed in the Constitution of India, is the chief of the
government, chief adviser to the president, head of the council of ministers and the leader of
the majority party in the parliament. The prime minister leads the executive of the
Government of India.

For Your Information : The President is the head of the State

IASbaba
Web: http://ilp.iasbaba.com/ Score:
Email: ilp@iasbaba.com 0.00 / 150
Page 25
Exam Title : 2019 - Test 1-Polity & Curr...
Email : rahulsreedhar8787@gmail.com
Contact :

Note: When ‘state’ is written with small ‘s’, it represents an area i.e. state as in Assam,
Karnataka etc. While when it is written with capital ‘S’, it refers to the country.

QUESTION 32. MTczODc4K1JhaHVsIFNyZWVkaGFyK3JhaHVsc3JlZWRoYXI4Nzg3QGdtYWlsLmNvbStRVUVTV


ElPTiAzMQ==
In which of the following situations can a President use his power of discretion?

1. To declare a National Emergency in the Country

2. To send the advice of Council of Minister back for reconsideration

3. To Pocket Veto an ordinary bill passed by the Parliament

4. To appoint the Prime Minister when there are more than one leaders contending to get the
support of majority

Select the code from below:

a) 1,2 and 3
b) 2,3 and 4
c) 1,3 and 4
d) All of the above
Correct Answer: B

Explanation

Solution (b)

There are at least three situations where the President can exercise the powers using his or her
own discretion.

· In the first place, we have already noted that the President can send back the advice given by
the Council of Ministers and ask the Council to reconsider the decision. In doing this, the
President acts on his (or her) own discretion. It is done when the President thinks that the
advice has certain flaws or legal lacunae, or that it is not in the best interests of the country.

· Secondly, the President also has veto power by which he can withhold or refuse to give assent
to Bills (other than Money Bill) passed by the Parliament.

· The third kind of discretion arises more out of political circumstances. Formally, the President
appoints the Prime Minister. Normally, in the parliamentary system, a leader who has the
support of the majority in the Lok Sabha would be appointed as Prime Minister and the
question of discretion would not arise. But imagine a situation when after an election, no leader
has a clear majority in the Lok Sabha. Imagine further that after attempts to forge alliances,
two or three leaders are claiming that they have the support of the majority in the house. Now,
the President has to decide whom to appoint as the Prime Minister. In such a situation, the
President has to use his own discretion in judging who really may have the support of the
majority or who can actually form and run the government.

Note: To declare a National Emergency a written approval of the Cabinet is required.

IASbaba
Web: http://ilp.iasbaba.com/ Score:
Email: ilp@iasbaba.com 0.00 / 150
Page 26
Exam Title : 2019 - Test 1-Polity & Curr...
Email : rahulsreedhar8787@gmail.com
Contact :

QUESTION 33. MTczODc4K1JhaHVsIFNyZWVkaGFyK3JhaHVsc3JlZWRoYXI4Nzg3QGdtYWlsLmNvbStRVUVTV


ElPTiAzMg==
Consider the following statements:

1. Prime Minister allocates the ranks and portfolios to the Ministers

2. A person can become a Minister even when he/she is not a Member of Parliament.

Which of the above statements are correct?

a) 1 only
b) 2 only
c) Both 1 and 2
d) Neither 1 nor 2
Correct Answer: C

Explanation

Solution (c)

The Prime Minister then decides who will be the ministers in the Council of Ministers. The
Prime Minister allocates ranks and portfolios to the ministers. Depending upon the seniority
and political importance, the ministers are given the ranks of cabinet minister, minister of State
or deputy minister. In the same manner, Chief Ministers of the States choose ministers from
their own party or coalition. The Prime Minister and all the ministers have to be members of the
Parliament. If someone becomes a minister or Prime Minister without being an MP, such a perso
n has to get elected to the Parliament within six months.

Think

Think of a Prime Minister who got the post when he was not a MoP .

QUESTION 34. MTczODc4K1JhaHVsIFNyZWVkaGFyK3JhaHVsc3JlZWRoYXI4Nzg3QGdtYWlsLmNvbStRVUVTV


ElPTiAzMw==
Which of the following statements are correct?

1. The Council of Minister is collectively responsible to the Lok Sabha.

2. A vote of no confidence even against a single minister leads to the resignation of the entire
Council of Ministers.

Select the code from following:

a) 1 only
b) 2 only
c) Both 1 and 2
d) Neither 1 nor 2
Correct Answer: C

IASbaba
Web: http://ilp.iasbaba.com/ Score:
Email: ilp@iasbaba.com 0.00 / 150
Page 27
Exam Title : 2019 - Test 1-Polity & Curr...
Email : rahulsreedhar8787@gmail.com
Contact :

Explanation

Solution (c)

The Council of Ministers is collectively responsible to the Lok Sabha. This provision means that
a Ministry which loses confidence of the Lok Sabha is obliged to resign. The principle indicates
that the ministry is an executive committee of the Parliament and it collectively governs on
behalf of the Parliament. Collective responsibility is based on the principle of the solidarity of
the cabinet . It implies that a vote of no confidence even against a single minister leads
to the resignation of the entire Council of Ministers . It also indicates that if a minister
does not agree with a policy or decision of the cabinet, he or she must either accept the
decision or resign. It is binding on all ministers to pursue or agree to a policy for which there is
collective responsibility.

Think

· No confidence Motion

· Censure Motion

QUESTION 35. MTczODc4K1JhaHVsIFNyZWVkaGFyK3JhaHVsc3JlZWRoYXI4Nzg3QGdtYWlsLmNvbStRVUVTV


ElPTiAzNA==
Which of the following Civil Services are part of All India Services?

1. Indian Administrative Service

2. Indian Police Service

3. Indian Revenue Service

4. Indian Foreign Service

Select the code from following:

a) 1 and 2
b) 1,2 and 3
c) 1,2 and 4
d) All of the above
Correct Answer: A
Explanation

Solution (a)

There are three All India Services –

1. Indian Administrative Service

2. Indian Police Service

3. Indian Forest Service

IASbaba
Web: http://ilp.iasbaba.com/ Score:
Email: ilp@iasbaba.com 0.00 / 150
Page 28
Exam Title : 2019 - Test 1-Polity & Curr...
Email : rahulsreedhar8787@gmail.com
Contact :

Indian Foreign Service and Indian Revenue Service are a part of Central Services of India.

QUESTION 36. MTczODc4K1JhaHVsIFNyZWVkaGFyK3JhaHVsc3JlZWRoYXI4Nzg3QGdtYWlsLmNvbStRVUVTV


ElPTiAzNQ==
Which of the following are the characteristics of the ‘All India Services’?

1. They are appointed by the Central Government while they work under the supervision of
State Government.

2. State Government cannot take any form of disciplinary action against them.

Select the code from below:

a) 1 only
b) 2 only
c) Both 1 and 2
d) Neither 1 nor 2
Correct Answer: A

Explanation

Solution (a)

An IAS or IPS officer is assigned to a particular State, where he or she works under the
supervision of the State government. However, the IAS or IPS officers are appointed by the
central government, they can go back into the service of the central government and most
importantly, only the central government can take disciplinary action against them. This means
that the key administrative officers of the States are under the supervision and control of the
central government.

Note: The above excerpt has been taken from Class XI NCERT. Here the word ‘Only’ mentioned
above is a sweeping remark so be cautious.

The state government can take disciplinary action against IAS, IPS officers serving
under them under Rule 7 (1) (b) of the All India Services (Discipline and Appeal)
Rules, 1969.

However, the penalty of dismissal, removal or compulsory retirement shall not be imposed on a
member of the service except by an order of the central government.

QUESTION 37. MTczODc4K1JhaHVsIFNyZWVkaGFyK3JhaHVsc3JlZWRoYXI4Nzg3QGdtYWlsLmNvbStRVUVTV


ElPTiAzNg==
Which of the following states in India have bicameral legislature?

1. Jammu and Kashmir

2. Karnataka

3. Andhra Pradesh

IASbaba
Web: http://ilp.iasbaba.com/ Score:
Email: ilp@iasbaba.com 0.00 / 150
Page 29
Exam Title : 2019 - Test 1-Polity & Curr...
Email : rahulsreedhar8787@gmail.com
Contact :

4. Uttar Pradesh

Select the code from below:

a) 1 and 4
b) 2 and 4
c) 2,3 and 4
d) All of the above
Correct Answer: D

Explanation

Solution (d)

The Parliament in India has two houses. When there are two houses of the legislature, it is
called a bicameral legislature. The two Houses of the Indian Parliament are the Council of
States or the Rajya Sabha and the House of the People or the Lok Sabha. The Constitution has
given the States the option of establishing either a unicameral or bicameral legislature. At
present only seven States have a bicameral legislature.

States having a bicameral legislature:

· Andhra Pradesh

· Bihar

· Jammu and Kashmir

· Karnataka

· Maharashtra

· Telangana

· Uttar Pradesh

IASbaba
Web: http://ilp.iasbaba.com/ Score:
Email: ilp@iasbaba.com 0.00 / 150
Page 30
Exam Title : 2019 - Test 1-Polity & Curr...
Email : rahulsreedhar8787@gmail.com
Contact :

QUESTION 38. MTczODc4K1JhaHVsIFNyZWVkaGFyK3JhaHVsc3JlZWRoYXI4Nzg3QGdtYWlsLmNvbStRVUVTV


ElPTiAzNw==
Rajya Sabha is the Upper House of the Parliament which represents the States of India. Which
of the following statements are correct regarding membership of Rajya Sabha?

IASbaba
Web: http://ilp.iasbaba.com/ Score:
Email: ilp@iasbaba.com 0.00 / 150
Page 31
Exam Title : 2019 - Test 1-Polity & Curr...
Email : rahulsreedhar8787@gmail.com
Contact :

1. Members of Rajya Sabha are directly elected.

2. Members of Rajya Sabha are elected for tenure of 5 years.

3. Every State has equal representation in Rajya Sabha.

Select the code from following:

a) 1 and 2
b) 2 and 3
c) 3 only
d) None of the above
Correct Answer: D

Explanation

Solution (d)

The Rajya Sabha represents the States of India. It is an indirectly elected body . Residents of
the State elect members to State Legislative Assembly. The elected members of State
Legislative Assembly in turn elect the members of the Rajya Sabha.

States with larger population get more representatives than States with smaller
population get. Thus, a more populous State like Uttar Pradesh sends 31 members to the
Rajya Sabha, while a smaller and less populous State like Sikkim has one seat in the Rajya
Sabha. Members of the Rajya Sabha are elected for a term of six years. They can get re-
elected.

Note: Rajya Sabha is the permanent house of the Parliament which cannot be dissolved.

QUESTION 39. MTczODc4K1JhaHVsIFNyZWVkaGFyK3JhaHVsc3JlZWRoYXI4Nzg3QGdtYWlsLmNvbStRVUVTV


ElPTiAzOA==
Which of the following functions are performed by the Legislature of India?

1. Financial Function

2. Constituent Function

3. Electoral Function

4. Judicial Function

Select the code from following:

a) 1 and 2
b) 2 and 3
c) 1,3 and 4
d) All of the above
Correct Answer: D
Explanation

IASbaba
Web: http://ilp.iasbaba.com/ Score:
Email: ilp@iasbaba.com 0.00 / 150
Page 32
Exam Title : 2019 - Test 1-Polity & Curr...
Email : rahulsreedhar8787@gmail.com
Contact :

Solution (d)

Functions of the Parliament:

Legislative Function: The Parliament enacts legislations for the country. Despite being the
chief law making body, the Parliament often merely approves legislations.

Control of Executive and ensuring its accountability: Perhaps the most vital function of the
Parliament is to ensure that the executive does not overstep its authority and remains
responsible to the people who have elected them.

Financial Function: legislature controls taxation and the way in which money is used by the
government. If the Government of India proposes to introduce any new tax, it has to get the
approval of the Lok Sabha. The financial powers of the Parliament involve grant of resources to
the government to implement its programmes . The government has to give an account to the
legislature about the money it has spent and resources that it wishes to raise. The legislature
also ensures that the government does not misspend or overspend. This is done through the
budget and annual financial statements.

Representation: Parliament represents the divergent views of members from different regiona
l, social, economic, religious groups of different parts of the country.

Debating Function: The Parliament is the highest forum of debate in the country. There is no
limitation on its power of discussion. Members are free to speak on any matter without fear.

Constituent Function: The Parliament has the power of discussing and enacting changes to
the Constitution. The constituent powers of both the houses are similar. All constitutional
amendments have to be approved by a special majority of both Houses.

Electoral functions: The Parliament also performs some electoral functions. It elects the
President and Vice President of India.

Judicial functions: The judicial functions of the Parliament include considering the proposals
for removal of President, Vice-President and Judges of High Courts and Supreme Court.

QUESTION 40. MTczODc4K1JhaHVsIFNyZWVkaGFyK3JhaHVsc3JlZWRoYXI4Nzg3QGdtYWlsLmNvbStRVUVTV


ElPTiAzOQ==
The term Federalism refers to

a) Presence of two houses of Parliament


b) Presence of a Parliamentary form of a government
c) Presence of dual polity in a country at the central and state level
d) Presence of single citizenship in a country
Correct Answer: C
Explanation

Solution (c)

IASbaba
Web: http://ilp.iasbaba.com/ Score:
Email: ilp@iasbaba.com 0.00 / 150
Page 33
Exam Title : 2019 - Test 1-Polity & Curr...
Email : rahulsreedhar8787@gmail.com
Contact :

Essentially, federalism is an institutional mechanism to accommodate two sets of polities — one


at the regional level and the other at the national level. Each government is autonomous in its
own sphere.

In some federal countries, there is even a system of dual citizenship. India has only a single
citizenship.

Think

· Bicameralism

· Diarchy

QUESTION 41. MTczODc4K1JhaHVsIFNyZWVkaGFyK3JhaHVsc3JlZWRoYXI4Nzg3QGdtYWlsLmNvbStRVUVTV


lPTiA0MA==
In the Constitution, it is defined “India, that is, Bharath as a union of states”. What does this
connote?

1. It talks about the name of the country as well as type of the Indian polity.

2. It reaffirms the idea of indestructible nature of Indian federation.

Choose the correct code

a) 1 only
b) 2 only
c) Both 1 and 2
d) None of the above
Correct Answer: C

Explanation

Solution (c)

Article 1 of the Indian constitution describes India, that is, Bharat as a ‘Union of States’ rather
than a ‘Federation of States.

This provision deals with two things: one, name of the country , and two, type of polity .

Secondly, the country is described as ‘Union’ although its Constitution is federal in


structure.

According to Dr B R Ambedkar, the phrase ‘Union of States’ has been preferred to ‘Federation
of States’ for two reasons: one, the Indian Federation is not the result of an agreement
among the states like the American Federation and two, the states have no right to secede
from the federation. The federation is a union because it is indestructible. The country is
an integral whole and divided into different states only for the convenience of administration.

IASbaba
Web: http://ilp.iasbaba.com/ Score:
Email: ilp@iasbaba.com 0.00 / 150
Page 34
Exam Title : 2019 - Test 1-Polity & Curr...
Email : rahulsreedhar8787@gmail.com
Contact :

Do you know?

There was no unanimity in the Constituent Assembly with regard to the name of the country.
Some members suggested the traditional name (Bharath) while other advocated the modern
name (India).

Hence, the Constituent Assembly had to adopt a mix of both (‘India, that is, Bharat’)

QUESTION 42. MTczODc4K1JhaHVsIFNyZWVkaGFyK3JhaHVsc3JlZWRoYXI4Nzg3QGdtYWlsLmNvbStRVUVTV


lPTiA0MQ==
Which of the following comprise the territory of India?

1. Territories of the states

2. Only those Union territories which have legislative assemblies.

3. Territories that may be acquired by government of India at any time

Choose the correct code

a) 1 and 2 only
b) 1 and 3 only
c) 2 and 3 only
d) All of the above
Correct Answer: B

Explanation

solution (b)

According to Article 1, the territory of India can be classified into three categories:

1. Territories of the states

2. Union territories

3. Territories that may be acquired by the Government of India at any time.

The names of states and union territories and their territorial extent are mentioned in the first
schedule of the Constitution. At present, there are 29 states and 7 union territories . The
provisions of the Constitution pertaining to the states are applicable to all the states (except
Jammu and Kashmir) in the same manner.

Do you know?

‘ Territory of India’ is a wider expression than the ‘Union of India’ because the latter
includes only states while the former includes not only the states but also union territories and
territories that may be acquired by the Government of India at any future time.

The states are the members of the federal system and share a distribution of powers
with the Centre. The union territories and the acquired territories, on the other hand, are
directly administered by the Central government.

IASbaba
Web: http://ilp.iasbaba.com/ Score:
Email: ilp@iasbaba.com 0.00 / 150
Page 35
Exam Title : 2019 - Test 1-Polity & Curr...
Email : rahulsreedhar8787@gmail.com
Contact :

Being a sovereign state, India can acquire foreign territories according to the modes recognised
by international law, i.e., cession (following treaty, purchase, gift, lease or plebiscite),
occupation (hitherto unoccupied by a recognised ruler), conquest or subjugation. For example,
India acquired several foreign territories such as Dadra and Nagar Haveli; Goa, Daman and
Diu; Puducherry; and Sikkim since the commencement of the Constitution.

QUESTION 43. MTczODc4K1JhaHVsIFNyZWVkaGFyK3JhaHVsc3JlZWRoYXI4Nzg3QGdtYWlsLmNvbStRVUVTV


ElPTiA0Mg==
Consider the following statements regarding the relationship of states and union territories
with the centre.

1. The names and territorial extent of states are mentioned in the constitution whereas the
same is not true with the union territories.

2. The states are the members of federal system whereas union territories are not.

3. The states share distribution of power with the centre but union territories are directly
administered by the centre.

Choose the correct code

a) 1 and 2 only
b) 2 and 3 only
c) 1 and 3 only
d) All of the above
Correct Answer: B

Explanation

Solution (b)

The names of states and union territories and their territorial extent are mentioned in
the first schedule of the Constitution. At present, there are 29 states and 7 union territories.
The provisions of the Constitution pertaining to the states are applicable to all the states
(except Jammu and Kashmir) in the same manner.

The states are the members of the federal system and share a distribution of powers
with the Centre . The union territories and the acquired territories, on the other hand, are dir
ectly administered by the Central government.

QUESTION 44. MTczODc4K1JhaHVsIFNyZWVkaGFyK3JhaHVsc3JlZWRoYXI4Nzg3QGdtYWlsLmNvbStRVUVTV


lPTiA0Mw==
Consider the following statements about the preamble of Indian constitution.

1. Preamble was enacted after the rest of the constitution was already made, as it was
considered less important by the constituent assembly.

2. Preamble cannot be amended because it contains the fundamental features of the Indian
constitution.

IASbaba
Web: http://ilp.iasbaba.com/ Score:
Email: ilp@iasbaba.com 0.00 / 150
Page 36
Exam Title : 2019 - Test 1-Polity & Curr...
Email : rahulsreedhar8787@gmail.com
Contact :

3. Preamble is of great importance as it acts as the source of power to legislature.

Choose the correct code

a) 1 only
b) 2 and 3 only
c) 3 only
d) None of the above
Correct Answer: D

Explanation

Solution (d)

Like any other part of the Constitution, the Preamble was also enacted by the Constituent
Assembly, but, after the rest of the Constitution was already enacted. The reason for
inserting the Preamble at the end was to ensure that it was in conformity with the
Constitution as adopted by the Constituent Assembly. While forwarding the Preamble for
votes, the president of the Constituent Assembly said, ‘The question is that Preamble stands
part of the Constitution’. The motion was then adopted.

Hence, the current opinion held by the Supreme Court that the Preamble is a part of the
Constitution, is in consonance with the opinion of the founding fathers of the Constitution.

However, two things should be noted:

1. The Preamble is neither a source of power to legislature nor a prohibition upon the
powers of legislature.

2. It is non-justiciable , that is, its provisions are not enforceable in courts of law

Do you know?

The question as to whether the Preamble can be amended under Article 368 of the Constitution
arose for the first time in the historic case of Kesavananda Bharati (1973) . It was urged
that the Preamble cannot be amended as it is not a part of the Constitution. The petitioner
contended that the amending power in Article 368 cannot be used to destroy or damage the
basic elements or the fundamental features of the Constitution , which are enshrined in
the Preamble.

The Supreme Court, however, held that the Preamble is a part of the Constitution. The Court
stated that the opinion tendered by it in the Berubari Union (1960) in this regard was
wrong, and held that the Preamble can be amended, subject to the condition that no
amendment is done to the ‘basic features’. In other words, the Court held that the basic eleme
nts or the fundamental features of the Constitution as contained in the Preamble
cannot be altered by an amendment under Article 368.

The Preamble has been amended only once so far , in 1976, by the 42nd Constitutional
Amendment Act, which has added three new words—Socialist, Secular and Integrity—to the
Preamble. This amendment was held to be valid.

IASbaba
Web: http://ilp.iasbaba.com/ Score:
Email: ilp@iasbaba.com 0.00 / 150
Page 37
Exam Title : 2019 - Test 1-Polity & Curr...
Email : rahulsreedhar8787@gmail.com
Contact :

QUESTION 45. MTczODc4K1JhaHVsIFNyZWVkaGFyK3JhaHVsc3JlZWRoYXI4Nzg3QGdtYWlsLmNvbStRVUVTV


ElPTiA0NA==
The Indian constitution promotes the feeling of fraternity through which of the following
instruments?

1. Single citizenship

2. Fundamental rights

3. Fundamental duties

4. Directive principles of state policy

5. Preamble

Choose the correct code

a) 1,2 ,3 and 4 only


b) 1,2,3 and 5 only
c) 1,2,4 and 5 only
d) All of the above
Correct Answer: D

Explanation

Solution (d)

Fraternity means a sense of brotherhood. The Constitution promotes this feeling of fraternity by
the system of single citizenship . Also, the Fundamental Duties (Article 51-A) say that it
shall be the duty of every citizen of India to promote harmony and the spirit of common
brotherhood amongst all the people of India transcending religious, linguistic, regional or
sectional diversities.

The Preamble declares that fraternity has to assure two things —the dignity of the
individual and the unity and integrity of the nation. The word ‘integrity’ has been added to the

preamble by the 42 nd Constitutional Amendment (1976)


According to K M Munshi, a member of the Drafting Committee of the Constituent Assembly,
the phrase ‘dignity of the individual’ signifies that the Constitution not only ensures material
betterment and maintains a democratic set-up, but that it also recognises that the personality of
every individual is sacred. This is highlighted through some of the provisions of the
Fundamental Rights and Directive Principles of State Policy, which ensure the dignity
of individuals . Further, the Fundamental Duties (Article 51A) also protect the dignity of
women by stating that it shall be the duty of every citizen of India to renounce practices
derogatory to the dignity of women, and also makes it the duty of every citizen of India to
uphold and protect the sovereignty, unity and integrity of India

Do you know?

The phrase ‘unity and integrity of the nation’ embraces both the psychological and territorial
dimensions of national integration. Article 1 of the Constitution describes India as a ‘Union of
States’ to make it clear that the states have no right to secede from the Union, implying the
indestructible nature of the Indian Union. It aims at overcoming hindrances to national
integration like communalism, regionalism, casteism, linguism and secessionism and so on.

IASbaba
Web: http://ilp.iasbaba.com/ Score:
Email: ilp@iasbaba.com 0.00 / 150
Page 38
Exam Title : 2019 - Test 1-Polity & Curr...
Email : rahulsreedhar8787@gmail.com
Contact :

QUESTION 46. MTczODc4K1JhaHVsIFNyZWVkaGFyK3JhaHVsc3JlZWRoYXI4Nzg3QGdtYWlsLmNvbStRVUVTV


ElPTiA0NQ==
Political equality in India is achieved through which of the following provisions of the
constitution?

1. No person is to be declared ineligible for inclusion in electoral rolls on grounds of religion,


race, caste or sex.

2. Elections to the Lok Sabha and the state assemblies to be on the basis of adult suffrage.

3. Abolition of titles

4. Equality of opportunity in the matters of public employment

Choose the correct code

a) 1 and 2 only
b) 1 and 3 only
c) 1,2 and 4 only
d) 2,3 and 4 only
Correct Answer: A

Explanation

Solution (a)

The term ‘equality’ means the absence of special privileges to any section of the society, and the
provision of adequate opportunities for all individuals without any discrimination. There are two
provisions in the Constitution that seek to achieve political equality. One, no person is to be
declared ineligible for inclusion in electoral rolls on grounds of religion, race, caste or
sex (Article 325). Two, elections to the Lok Sabha and the state assemblies to be on the
basis of adult suffrage (Article 326).

The Directive Principles of State Policy (Article 39) secures to men and women equal right to an
adequate means of livelihood and equal pay for equal work.

Do you know?

The Preamble secures to all citizens of India equality of status and opportunity. This provision
embraces three dimensions of equality—civic, political and economic.

The following provisions of the chapter on Fundamental Rights ensure civic equality:

(a) Equality before the law (Article 14).

(b) Prohibition of discrimination on grounds of religion, race, caste, sex or place of birth (Article

15).

(c) Equality of opportunity in matters of public employment (Article 16).

(d) Abolition of untouchability (Article 17).

IASbaba
Web: http://ilp.iasbaba.com/ Score:
Email: ilp@iasbaba.com 0.00 / 150
Page 39
Exam Title : 2019 - Test 1-Polity & Curr...
Email : rahulsreedhar8787@gmail.com
Contact :

(e) Abolition of titles (Article 18).

QUESTION 47. MTczODc4K1JhaHVsIFNyZWVkaGFyK3JhaHVsc3JlZWRoYXI4Nzg3QGdtYWlsLmNvbStRVUVTV


ElPTiA0Ng==
Which of the following correctly explains the idea of liberty enshrined in the Indian
constitution?

a) Liberty is the absence of restraints on the activities of individuals


b) Liberty means license to do what one likes
c) Liberty conceived by the Preamble does not have any limitations hence it is absolute but
not qualified
d) Violation of one’s liberty is not enforceable through any courts in India
Correct Answer: A

Explanation

Solution (a)

The term ‘liberty’ means the absence of restraints on the activities of individuals, and at
the same time, providing opportunities for the development of individual personalities.

The Preamble secures to all citizens of India liberty of thought, expression, belief, faith and
worship, through their Fundamental Rights , enforceable in court of law, in case of
violation.

Liberty as elaborated in the Preamble is very essential for the successful functioning of the
Indian democratic system. However, liberty does not mean ‘license’ to do what one likes, a
nd has to be enjoyed within the limitations mentioned in the Constitution itself. In brief, the
liberty conceived by the Preamble or fundamental rights is not absolute but qualified .

Do you know?

The ideals of liberty, equality and fraternity in our Preamble have been taken from the French
Revolution (1789–1799).

QUESTION 48. MTczODc4K1JhaHVsIFNyZWVkaGFyK3JhaHVsc3JlZWRoYXI4Nzg3QGdtYWlsLmNvbStRVUVTV


ElPTiA0Nw==
The term distributive justice in the Indian context comprises of which of the following?

a) Social justice and economic justice


b) Social justice and political justice
c) Political justice and economic justice
d) None of the above
Correct Answer: A
Explanation

IASbaba
Web: http://ilp.iasbaba.com/ Score:
Email: ilp@iasbaba.com 0.00 / 150
Page 40
Exam Title : 2019 - Test 1-Polity & Curr...
Email : rahulsreedhar8787@gmail.com
Contact :

Solution (a)

The term ‘justice’ in the Preamble embraces three distinct forms—social, economic and
political, secured through various provisions of Fundamental Rights and Directive
Principles.

Social justice denotes the equal treatment of all citizens without any social distinction ba
sed on caste, colour, race, religion, sex and so on. It means absence of privileges being
extended to any particular section of the society , and improvement in the conditions of
backward classes (SCs, STs and OBCs) and women.

Economic justice denotes the non-discrimination between people on the basis of


economic factors. It involves the elimination of glaring in-equalities in wealth, income and
property. A combination of social justice and economic justice denotes what is known as
‘distributive justice’.

Political justice implies that all citizens should have equal political rights, equal access to
all political offices and equal voice in the government.

Do you know?

The ideal of justice—social, economic and political—has been taken from the Russian
Revolution (1917)

QUESTION 49. MTczODc4K1JhaHVsIFNyZWVkaGFyK3JhaHVsc3JlZWRoYXI4Nzg3QGdtYWlsLmNvbStRVUVTV


ElPTiA0OA==
Indian polity is considered to be a republic, what does this connote?

a) The head of the state in India enjoys hereditary position


b) The head of the government in India is indirectly elected
c) The head of the state in India is elected
d) The head of the state in India is directly elected
Correct Answer: C

Explanation

Solution (c)

A democratic polity can be classified into two categories— monarchy and republic . In a mo
narchy , the head of the state (usually king or queen) enjoys a hereditary position , that is,
he comes into office through succession, example Britain. In a republic, on the other hand, the
head of the state is always elected directly or indirectly for a fixed period, example
USA.

Therefore, the term ‘republic’ in our Preamble indicates that India has an elected head
called the president. He is elected indirectly for a fixed period of five years.

Do you know?

IASbaba
Web: http://ilp.iasbaba.com/ Score:
Email: ilp@iasbaba.com 0.00 / 150
Page 41
Exam Title : 2019 - Test 1-Polity & Curr...
Email : rahulsreedhar8787@gmail.com
Contact :

A republic also means two more things: one, vesting of political sovereignty in the people
and not in a single individual like a king ; second, the absence of any privileged class an
d hence all public offices being opened to every citizen without any discrimination.

QUESTION 50. MTczODc4K1JhaHVsIFNyZWVkaGFyK3JhaHVsc3JlZWRoYXI4Nzg3QGdtYWlsLmNvbStRVUVTV


ElPTiA0OQ==
Which of the following are considered to be the instruments of direct democracy?

1. Referendum

2. Initiative

3. Recall

4. Plebiscite

Choose the correct code

a) 1,2 and 3 only


b) 1,3 and 4 only
c) 2,3 and 4 only
d) All of the above
Correct Answer: D

Explanation

Solution (d)

A democratic polity, as stipulated in the Preamble, is based on the doctrine of popular


sovereignty, that is, possession of supreme power by the people.

Democracy is of two types—direct and indirect. In direct democracy, the people exercise their
supreme power directly as is the case in Switzerland. There are four devices of direct
democracy, namely, Referendum, Initiative, Recall and Plebiscite . In indirect democracy,
on the other hand, the representatives elected by the people exercise the supreme power and
thus carry on the government and make the laws. This type of democracy, also known as
representative democracy , is of two kinds— parliamentary and presidential .

The Indian Constitution provides for representative parliamentary democracy under


which the executive is responsible to the legislature for all its policies and actions. Universal
adult franchise, periodic elections, rule of law, independence of judiciary, and absence of
discrimination on certain grounds are the manifestations of the democratic character of the
Indian polity.

The term ‘democratic’ is used in the Preamble in the broader sense embracing not only politica
l democracy but also social and economic democracy .

QUESTION 51. MTczODc4K1JhaHVsIFNyZWVkaGFyK3JhaHVsc3JlZWRoYXI4Nzg3QGdtYWlsLmNvbStRVUVTV


lPTiA1MA==

IASbaba
Web: http://ilp.iasbaba.com/ Score:
Email: ilp@iasbaba.com 0.00 / 150
Page 42
Exam Title : 2019 - Test 1-Polity & Curr...
Email : rahulsreedhar8787@gmail.com
Contact :

Which of the following are examples of Voice over Internet Protocol (VoIP)?

1. WhatsApp Calling

2. Skype

3. Facebook Video and Voice Chat

Select the correct code:

a) 1 Only
b) 1 and 2
c) 2 and 3
d) All of the above
Correct Answer: D

Explanation

Solution (d)

All the options are correct.

Voice over Internet Protocol is a category of hardware and software that enables people to
use the Internet as the transmission medium for telephone calls by sending voice data in
packets using IP rather than by traditional circuit transmissions of the PSTN.

THINK!

· BSNL Wings

Source: https://www.livemint.com/Industry/CNdkUyv3l46Hkr8pCdqgBK/What-to-
expect-from-BSNL-Wings-Indias-first-internet-calli.html

QUESTION 52. MTczODc4K1JhaHVsIFNyZWVkaGFyK3JhaHVsc3JlZWRoYXI4Nzg3QGdtYWlsLmNvbStRVUVTV


lPTiA1MQ==
Consider the following statements about Harmonized System (HS)

1. It allows classification of traded goods on a common basis for customs purposes

2. It was developed under the Maastricht Treaty when an economic and customs union of 28
European countries was formed

Select the correct statements

a) 1 Only
b) 2 Only
c) Both 1 and 2
d) Neither 1 nor 2
Correct Answer: A
Explanation

IASbaba
Web: http://ilp.iasbaba.com/ Score:
Email: ilp@iasbaba.com 0.00 / 150
Page 43
Exam Title : 2019 - Test 1-Polity & Curr...
Email : rahulsreedhar8787@gmail.com
Contact :

Solution (a)

Harmonized Commodity Description and Coding System , also known as the Harmonized
System (HS) is an international nomenclature for the classification of products. It allows
participating countries to classify traded goods on a common basis for customs purposes. At the
international level, the Harmonized System (HS) for classifying goods is a six-digit code system.

It has been developed and maintained by the World Customs Organization (WCO)

Source: http://www.forbesindia.com/blog/economy-policy/utilising-free-trade-
agreements-for-what-they-mean-to-business/

QUESTION 53. MTczODc4K1JhaHVsIFNyZWVkaGFyK3JhaHVsc3JlZWRoYXI4Nzg3QGdtYWlsLmNvbStRVUVTV


ElPTiA1Mg==
Consider the following statements about ‘Golden Jackal’

1. It enjoys the same level of protection as the Tiger

2. It is mainly found in the deserts and arid regions

Select the correct statements

a) 1 Only
b) 2 Only
c) Both 1 and 2
d) Neither 1 nor 2
Correct Answer: D
Explanation

Solution (d)

Golden Jackal

· It is a widespread species. It is fairly common throughout its range with high densities
observed in areas with abundant food and cover.

· A minimum population estimate of over 80,000 is estimated for the Indian sub-continent.

· They are abundant in valleys and beside rivers and their tributaries, canals, lakes

· They are rare in foothills and low mountains.

· Due to their tolerance of dry habitats and their omnivorous diet, the Golden Jackal can live in a
wide variety of habitats.

· IUCN Status – Least Concern

· It is included in CITES Appendix III (in India).

· They feature on Schedule III of the Wildlife Protection Act (1972) of India and are afforded the
least legal protection

IASbaba
Web: http://ilp.iasbaba.com/ Score:
Email: ilp@iasbaba.com 0.00 / 150
Page 44
Exam Title : 2019 - Test 1-Polity & Curr...
Email : rahulsreedhar8787@gmail.com
Contact :

Source: https://www.thehindu.com/news/national/andhra-pradesh/golden-jackal-faces-
threat-in-its-habitat/article24428393.ece

QUESTION 54. MTczODc4K1JhaHVsIFNyZWVkaGFyK3JhaHVsc3JlZWRoYXI4Nzg3QGdtYWlsLmNvbStRVUVTV


lPTiA1Mw==
‘Helsinki Accords’ is associated with

a) International Water Sharing


b) Protection of Human Rights
c) Climate Change
d) None of the above
Correct Answer: B
Explanation

Solution (b)

Helsinki Accords

· It is also known as the Helsinki Final Act, or Helsinki Declaration

· It were signed on August 1, 1975, at the conclusion of the first Conference on Security and Co-
operation in Europe (now called the OSCE or Organisation for Security and Co-operation in
Europe).

· It were made mainly to reduce tension between the Western blocs and the then Soviet Union
by securing their common acceptance of the post-Second World War status quo in Europe.

· It were signed by all European countries barring Albania which became its signatory in 1991
and the US and Canada.

· It recognised that the post-World War II frontiers in Europe were inviolable and all the 35
nations that signed it pledged to respect the human rights and fundamental freedoms besides
cooperating on various areas.

· They are not binding and do not have a treaty status.

DO YOU KNOW?

Organization for Security and Co-operation in Europe (OSCE)

· It is the world's largest security-oriented intergovernmental organization.

· Its mandate includes issues such as arms control, promotion of human rights, freedom of the
press, and fair elections.

THINK!

· Paris Principles

Source: http://www.newindianexpress.com/world/2018/jun/28/finnish-neutrality-key-to-
helsinki-hosting-us-russia-summit-1835204.html

IASbaba
Web: http://ilp.iasbaba.com/ Score:
Email: ilp@iasbaba.com 0.00 / 150
Page 45
Exam Title : 2019 - Test 1-Polity & Curr...
Email : rahulsreedhar8787@gmail.com
Contact :

QUESTION 55. MTczODc4K1JhaHVsIFNyZWVkaGFyK3JhaHVsc3JlZWRoYXI4Nzg3QGdtYWlsLmNvbStRVUVTV


ElPTiA1NA==
Consider the following statements about ‘ Samagra Shiksha’

1. It is under the aegis of Ministry of Human Resources Development

2. It is aimed at skill development of unemployed engineering graduates from tier 2 and tier 3
cities

3. Skill training would be done based on the National Skill Qualification Framework (NSQF)

Select the correct statements

a) 1 Only
b) 1 and 2
c) 2 and 3
d) None of the above
Correct Answer: A
Explanation

Solution (a)

Samagra Shiksha

· It is an overarching programme for the school education sector extending from pre-school to
class 12

· It has been prepared with the broader goal of improving school effectiveness measured in
terms of equal opportunities for schooling and equitable learning outcomes.

· It subsumes the three Schemes of Sarva Shiksha Abhiyan (SSA), Rashtriya Madhyamik
Shiksha Abhiyan (RMSA) and Teacher Education (TE).

· The shift in the focus is from project objectives to improving systems level performance and
schooling outcomes which will be the emphasis of the combined Scheme along-with
incentivizing States towards improving quality of education

· It envisages the ‘school’ as a continuum from pre-school, primary, upper primary, secondary to
Senior Secondary levels.

· The vision of the Scheme is to ensure inclusive and equitable quality education from pre-
school to senior secondary stage in accordance with the Sustainable Development Goal (SDG)
for Education.

· Concerned Ministry – HRD

QUESTION 56. MTczODc4K1JhaHVsIFNyZWVkaGFyK3JhaHVsc3JlZWRoYXI4Nzg3QGdtYWlsLmNvbStRVUVTV


ElPTiA1NQ==
Consider the following statements about Pradhan Mantri Jan Vikas Karyakram (PMJVK)

IASbaba
Web: http://ilp.iasbaba.com/ Score:
Email: ilp@iasbaba.com 0.00 / 150
Page 46
Exam Title : 2019 - Test 1-Polity & Curr...
Email : rahulsreedhar8787@gmail.com
Contact :

1. It is aimed at improving socio-economic conditions of the minorities and providing basic


amenities to them for improving quality of life of the people

2. It will be implemented by Khadi and Village Industries Commission (KVIC) across the country

Select the correct statements

a) 1 Only
b) 2 Only
c) Both 1 and 2
d) Neither 1 nor 2
Correct Answer: A

Explanation

Solution (a)

Pradhan Mantri Jan Vikas Karyakram (PMJVK)

· Concerned Ministry - Ministry of Minority Affairs

· Multi-sectoral Development Programme ( MsDP ) is renamed and restructured as PMJVK


during the remaining period of 14th Finance Commission.

· It will be implemented as a Centrally Sponsored Scheme

· A new area for implementation of PMJVK has been included namely Minority Concentration
District Headquarters (MCD Hq )

· It has used the selected socio-economic, basic amenities and population data of Census 2011
and have identified MCBs, MCTs and MCD Hqs .

· The identified areas namely MCBs, MCTs, MCD Hqs and COVs for implementation of PMJVK
will be known as Minority Concentration Areas (MCA)

· It aims at improving socio-economic conditions of the minorities and providing basic amenities
to them for improving quality of life of the people and reducing imbalances in the identified
minority concentration areas.

· The areas for implementation of PMJVK has been identified on the basis of substantial
population of minority communities i.e. 25% of minority population in the area and
backwardness parameters in the area concerned which are below the national average.

· The Karyakram will be implemented through State/Central agencies. The State may, however,
decide to execute the project through any qualified, reputed, experienced agency, including
renowned and widely accepted NGOs, justification for which should be mentioned in the
proposal. Implementation of project and operationalization of the assets may be allowed
through Public Private Partnership (PPP) mode wherever felt feasible.

DO YOU KNOW?

· At present six communities namely Muslims, Sikhs, Christians, Buddhists, Zoroastrians ( Parsi
s ) and Jains have been notified as Minority Communities under the National Commission for
Minorities Act, 1992.

IASbaba
Web: http://ilp.iasbaba.com/ Score:
Email: ilp@iasbaba.com 0.00 / 150
Page 47
Exam Title : 2019 - Test 1-Polity & Curr...
Email : rahulsreedhar8787@gmail.com
Contact :

· As per the Census 2011, the percentage of minorities in the country is about 19.3% of the total
population of the country. The population of Muslims are 14.2%; Christians 2.3%; Sikhs 1.7%,
Buddhists 0.7%, Jain 0.4% and Parsis 0.006%.

· Maharashtra, Gujrat and West Bengal accorded minority status to Jews.

THINK!

· USTTAD Scheme

· Jiyo Parsi Scheme

· Nai Roshni and Nai Manzil Scheme

QUESTION 57. MTczODc4K1JhaHVsIFNyZWVkaGFyK3JhaHVsc3JlZWRoYXI4Nzg3QGdtYWlsLmNvbStRVUVTV


ElPTiA1Ng==
Consider the following statements about Central Adoption Resource Authority (CARA)

1. It is under the aegis of Ministry of Women & Child Development

2. It deals with inter-country adoptions according to Hague Convention on Inter-country


Adoption

Select the correct statements

a) 1 Only
b) 2 Only
c) Both 1 and 2
d) Neither 1 nor 2
Correct Answer: C

Explanation

Solution (c)

Central Adoption Resource Authority (CARA) is a statutory body of Ministry of Women & Child
Development, Government of India. It functions as the nodal body for adoption of Indian
children and is mandated to monitor and regulate in-country and inter-country adoptions. CARA
is designated as the Central Authority to deal with inter-country adoptions in accordance with
the provisions of the Hague Convention on Inter-country Adoption, 1993, ratified by
Government of India in 2003.

CARA primarily deals with adoption of orphan, abandoned and surrendered children through its
associated / recognised adoption agencies

THINK!

· Hague Adoption Convention

· Hague Abduction Convention

IASbaba
Web: http://ilp.iasbaba.com/ Score:
Email: ilp@iasbaba.com 0.00 / 150
Page 48
Exam Title : 2019 - Test 1-Polity & Curr...
Email : rahulsreedhar8787@gmail.com
Contact :

QUESTION 58. MTczODc4K1JhaHVsIFNyZWVkaGFyK3JhaHVsc3JlZWRoYXI4Nzg3QGdtYWlsLmNvbStRVUVTV


ElPTiA1Nw==
Which of the following statements is the most appropriate explanation of ‘ FASTags ’?

a) Low-cost small satellite launcher by ISRO


b) Payment app using Unified Payments Interface (UPI)
c) RFID tag that enables automatic deduction of toll charges
d) Mechanism to achieve rural advancement through S&T interventions
Correct Answer: C
Explanation

Solution (c)

FASTags

· It is a device that can be installed on the windshield of any vehicle, and toll payments can be
made directly from the pre-paid account linked to it.

· Hence such vehicles do not have to stop at toll plazas for payment of fees.

· FASTags can be purchased online from the websites of banks, the National Highways
Authority of India and Indian Highways Management Company Limited.

· It employs Radio Frequency Identification (RFID) technology

THINK!

· Sukhad Yatra App by MoRTH

· RFID Technology

QUESTION 59. MTczODc4K1JhaHVsIFNyZWVkaGFyK3JhaHVsc3JlZWRoYXI4Nzg3QGdtYWlsLmNvbStRVUVTV


ElPTiA1OA==
Which one of the following is known as the Marshall Plan?

a) US military intervention in Vietnam


b) US Nuclear Strike on Japan
c) Iran Nuclear Deal
d) US post-war economic assistance to Europe
Correct Answer: D
Explanation

Solution (d)

Marshall Plan

IASbaba
Web: http://ilp.iasbaba.com/ Score:
Email: ilp@iasbaba.com 0.00 / 150
Page 49
Exam Title : 2019 - Test 1-Polity & Curr...
Email : rahulsreedhar8787@gmail.com
Contact :

· It is also known as the European Recovery Program

· It channelled over $13 billion to finance the economic recovery of Europe between 1948 and
1951.

· It successfully sparked economic recovery, meeting its objective of ‘restoring the confidence of
the European people in the economic future of their own countries and of Europe as a whole.’

· The plan is named for Secretary of State George C. Marshall, who announced it in a
commencement speech at Harvard University on June 5, 1947.

QUESTION 60. MTczODc4K1JhaHVsIFNyZWVkaGFyK3JhaHVsc3JlZWRoYXI4Nzg3QGdtYWlsLmNvbStRVUVTV


ElPTiA1OQ==
Consider the following statements about Generalized System of Preferences (GSP)

1. It is a framework under which developed countries give preferential tariff treatment to goods
imported from certain developing countries.

2. The Enabling Clause is the WTO legal basis for the Generalized System of Preferences (GSP)

Select the correct statements

a) 1 Only
b) 2 Only
c) Both 1 and 2
d) Neither 1 nor 2
Correct Answer: C

Explanation

Solution (c)

Generalised System of Preferences (GSP)

· It is a preferential tariff system extended by developed countries (also known as preference


giving countries or donor countries) to developing countries (also known as preference
receiving countries or beneficiary countries).

· It involves reduced MFN Tariffs or duty-free entry of eligible products exported by beneficiary
countries to the markets of donor countries.

Benefits

· Indian exporters benefit indirectly - through the benefit that accrues to the importer by way of
reduced tariff or duty free entry of eligible Indian products

· Reduction or removal of import duty on an Indian product makes it more competitive to the
importer - other things (e.g. quality) being equal.

· This tariff preference helps new exporters to penetrate a market and established exporters to
increase their market share and to improve upon the profit margins, in the donor country.

IASbaba
Web: http://ilp.iasbaba.com/ Score:
Email: ilp@iasbaba.com 0.00 / 150
Page 50
Exam Title : 2019 - Test 1-Polity & Curr...
Email : rahulsreedhar8787@gmail.com
Contact :

Enabling Clause

· The Enabling Clause officially called the “Decision on Differential and More Favourable Treat
ment, Reciprocity and Fuller Participation of Developing Countries” was adopted under GATT in
1979 and enables developed members to give differential and more favourable treatment to
developing countries.

· The Enabling Clause is the WTO legal basis for the Generalized System of Preferences (GSP).
Under the Generalized System of Preferences, developed countries offer non-reciprocal
preferential treatment (such as zero or low duties on imports) to products originating in
developing countries. Preference-giving countries unilaterally determine which countries and
which products are included in their schemes.

· The Enabling Clause is also the legal basis for regional arrangements among developing
countries and for the Global System of Trade Preferences (GSTP), under which a number of
developing countries exchange trade concessions among themselves.

THINK!

· MFN Status

QUESTION 61. MTczODc4K1JhaHVsIFNyZWVkaGFyK3JhaHVsc3JlZWRoYXI4Nzg3QGdtYWlsLmNvbStRVUVTV


lPTiA2MA==
Which one of the following proposed on theoretical grounds that the universe is expanding?

a) Georges Lemaitre
b) Abbe Barthelemy Carre
c) Sheldon Cooper
d) Harlow Shapley
Correct Answer: A
Explanation

Solution (a)

Georges Lemaitre

· He was a Belgian cosmologist who in 1927 propounded the Big Bang theory when he theorised
that the universe began as a single point and expanded to become as big as it is now. 173878

· Lemaitre’s thesis was based on calculations derived from Albert Einstein’s Theory of General
Relativity.

QUESTION 62. MTczODc4K1JhaHVsIFNyZWVkaGFyK3JhaHVsc3JlZWRoYXI4Nzg3QGdtYWlsLmNvbStRVUVTV


lPTiA2MQ==
Which of the following countries are members of the Asia-Pacific Trade Agreement (APTA)?

1. China

IASbaba
Web: http://ilp.iasbaba.com/ Score:
Email: ilp@iasbaba.com 0.00 / 150
Page 51
Exam Title : 2019 - Test 1-Polity & Curr...
Email : rahulsreedhar8787@gmail.com
Contact :

2. India

3. South Korea

4. Laos

Select the correct code:

a) 1, 3 and 4
b) 2, 3 and 4
c) 1, 2 and 3
d) All of the above
Correct Answer: D
Explanation

Solution (d)

Asia-Pacific Trade Agreement


(APTA)

· It aims to promote economic development through the adoption of mutually beneficial trade
liberalization measures

· It is a Preferential Trade Agreement, under which the basket of items as well as extent of tariff
concessions are enlarged during the trade negotiating rounds which are launched from time to
time.

· It was previously named the Bangkok Agreement

· It was signed in 1975 as an initiative of United Nations Economic and Social Commission for
Asia and the Pacific (ESCAP)

· APTA is the only operational trade agreement linking China and India

Membership

· Bangladesh

· China

· India

· Mongolia

· South Korea

· Laos

· Sri Lanka

Source: http://pib.nic.in/PressReleseDetail.aspx?PRID=1537323

IASbaba
Web: http://ilp.iasbaba.com/ Score:
Email: ilp@iasbaba.com 0.00 / 150
Page 52
Exam Title : 2019 - Test 1-Polity & Curr...
Email : rahulsreedhar8787@gmail.com
Contact :

QUESTION 63. MTczODc4K1JhaHVsIFNyZWVkaGFyK3JhaHVsc3JlZWRoYXI4Nzg3QGdtYWlsLmNvbStRVUVTV


ElPTiA2Mg==
‘ cVIGIL Mobile App’ is associated with

a) Soil Health
b) Model code of conduct during elections
c) Allocation of Coal Mines
d) Income tax
Correct Answer: B
Explanation

Solution (b)

cVIGIL Mobile App

· It was launched for citizens to report any violation of the model code of conduct during
elections.

· It will be operational only where elections are announced.

· It is a GIS-based mobile application

Model Code of Conduct

· It is a set of guidelines issued by the Election Commission of India for conduct of political
parties and candidates during elections mainly with respect to speeches, polling day, polling
booths, election manifestos, processions and general conduct.

· It comes into force immediately on announcement of the election schedule by the commission
for the need of ensuring free and fair elections.

Source: http://pib.nic.in/PressReleseDetail.aspx?PRID=1537492

QUESTION 64. MTczODc4K1JhaHVsIFNyZWVkaGFyK3JhaHVsc3JlZWRoYXI4Nzg3QGdtYWlsLmNvbStRVUVTV


lPTiA2Mw==
‘ Bru ’ tribe inhabits which of the following states?

a) Tamil Nadu
b) Odisha
c) Mizoram
d) Kerala
Correct Answer: C
Explanation

Solution (c)

Bru Tribe

IASbaba
Web: http://ilp.iasbaba.com/ Score:
Email: ilp@iasbaba.com 0.00 / 150
Page 53
Exam Title : 2019 - Test 1-Polity & Curr...
Email : rahulsreedhar8787@gmail.com
Contact :

News: A “historic agreement” had been signed among the governments of Mizoram and
Tripura and the Mizoram Bru Displaced People’s Forum, it brought to an end a 21-year wait for
over 32,000 Bru tribals , who had been displaced from Mizoram and were living in Tripura

About

· Centre will provide financial assistance for rehabilitation of Brus in Mizoram and address their
issues of security, education, livelihood etc. in consultation with Governments of Mizoram and
Tripura.

· Bru (or Reang ) tribals inhabit parts of some North-eastern states. In Mizoram, they are
largely restricted to Mamit and Kolasib districts.

THINK!

· Sixth Schedule of the Indian Constitution

Source: http://pib.nic.in/PressReleseDetail.aspx?PRID=1537513

QUESTION 65. MTczODc4K1JhaHVsIFNyZWVkaGFyK3JhaHVsc3JlZWRoYXI4Nzg3QGdtYWlsLmNvbStRVUVTV


ElPTiA2NA==
Sunil Mehta Committee is concerned with

a) Reforming Corporate Governance


b) Penetration of Digital Payments
c) Doubling Farmer’s Income
d) Faster resolution of stressed assets
Correct Answer: D
Explanation

Solution (d)

Sunil Mehta Committee

· It deals with faster resolution of stressed assets.

· For loans up to Rs 50 crore, it suggested a steering committee within the bank to resolve it
within 90 days.

· For loans of Rs 50-500 crore, it has suggested another bank-led resolution within 180 days.

· For loans above Rs 500 crore, it has suggested setting up an asset management company with
private participation.

· It suggested setting up an Alternative Investment Fund that will raise resources from banks
and institutional investors so that it can bid for the insolvent assets under insolvency and
bankruptcy.

· Asset trading platform for stressed assets.

THINK!

IASbaba
Web: http://ilp.iasbaba.com/ Score:
Email: ilp@iasbaba.com 0.00 / 150
Page 54
Exam Title : 2019 - Test 1-Polity & Curr...
Email : rahulsreedhar8787@gmail.com
Contact :

· Insolvency and Bankruptcy Code

Source: https://www.thehindu.com/business/Economy/committee-recommends-setting-
up-amcs-for-large-stressed-loans/article24314618.ece

QUESTION 66. MTczODc4K1JhaHVsIFNyZWVkaGFyK3JhaHVsc3JlZWRoYXI4Nzg3QGdtYWlsLmNvbStRVUVTV


ElPTiA2NQ==
Consider the following statements about WIPO Copyright Treaty

1. It deals with the protection of works and the rights of their authors in the digital environment

2. It ensures that computer programs are protected as literary works

Select the correct statements

a) 1 Only
b) 2 Only
c) Both 1 and 2
d) Neither 1 nor 2
Correct Answer: C

Explanation

Solution (c)

WIPO Copyright Treaty

News: Cabinet approves accession to WIPO Copyright Treaty, 1996

About

· It is a Special agreement under Berne Convention (for protection of literary and artistic
works).

· It deals with the protection of works and the rights of their authors in the digital environment.

· It has provisions to extend the protection of copyrights contained therein to the digital
environment.

· It recognises the rights specific to digital environment, of making work available, to address
"on-demand" and other interactive modes of access

· It ensures that computer programs are protected as literary works

As to the rights granted to authors, apart from the rights recognized by the Berne
Convention, the Treaty also grants:

· Right of distribution

· Right of rental

· Broader right of communication to the public

IASbaba
Web: http://ilp.iasbaba.com/ Score:
Email: ilp@iasbaba.com 0.00 / 150
Page 55
Exam Title : 2019 - Test 1-Polity & Curr...
Email : rahulsreedhar8787@gmail.com
Contact :

It provides framework for creators and right owners to use technical tools to protect their
works and safeguard information about their use i.e. Protection of Technological Protection
Measures (TPMs) and Rights Management Information (RMI).

THINK!

· TRIPS

· WIPO Performances and Phonograms Treaty

Source: http://pib.nic.in/PressReleseDetail.aspx?PRID=1537563

QUESTION 67. MTczODc4K1JhaHVsIFNyZWVkaGFyK3JhaHVsc3JlZWRoYXI4Nzg3QGdtYWlsLmNvbStRVUVTV


ElPTiA2Ng==
Consider the following statements about ‘Khan Prahari ’

1. It aims to curb instances of illegal mining activity

2. It is an interactive platform for all the stakeholders to compress the timelines for statutory
and other clearances related to mining

Select the correct statements

a) 1 Only
b) 2 Only
c) Both 1 and 2
d) Neither 1 nor 2
Correct Answer: A
Explanation

Solution (a)

Khan Prahari

· It is a tool for reporting any activity taking place related to illegal coal mining like rat hole
mining, pilferage etc.

· One can upload geo-tagged photographs of the incident along with textual information directly
to the system.

· Hence, both satellite data and human information will be used to capture information on the u
nauthorised mining activities.

THINK!

· TAMRA Portal

Source: http://pib.nic.in/PressReleseDetail.aspx?PRID=1537749

IASbaba
Web: http://ilp.iasbaba.com/ Score:
Email: ilp@iasbaba.com 0.00 / 150
Page 56
Exam Title : 2019 - Test 1-Polity & Curr...
Email : rahulsreedhar8787@gmail.com
Contact :

QUESTION 68. MTczODc4K1JhaHVsIFNyZWVkaGFyK3JhaHVsc3JlZWRoYXI4Nzg3QGdtYWlsLmNvbStRVUVTV


ElPTiA2Nw==
Recently, Government has declared Minimum Support Price (MSP) by adopting ‘A2+FL cost
formula’. The ‘A2+FL formula’ takes into account which of the following?

1. Hired labour

2. Cost of seeds

3. Imputed rent on land

Select the correct code:

a) 1 and 2
b) 2 and 3
c) 1 and 3
d) All of the above
Correct Answer: A

Explanation

Solution (a)

CACP has three different definitions of productions costs

· A2 formula is actual paid out cost which includes cost of seeds, fertilisers , pesticides, hired la
bour , fuel etc

· A2+FL formula takes into account actual cost plus imputed value of family labour in the
production of a crop.

· The C2 formula factors in a lot of costs, including imputed rent on land and interest on capital,
which makes the cost of production much higher than the level on which the Commission for
Agricultural Costs and Prices bases its recommendations.

· C2 > A2+FL > A2

THINK!

· Fair and Remunerative Price

Source: http://pib.nic.in/PressReleseDetail.aspx?PRID=1537930

QUESTION 69. MTczODc4K1JhaHVsIFNyZWVkaGFyK3JhaHVsc3JlZWRoYXI4Nzg3QGdtYWlsLmNvbStRVUVTV


ElPTiA2OA==
If you want to see ‘Salt Water Crocodile’ in their natural habitat, which one of the following is
the best place to visit?

a) Bhitarkanika National Park


b) Chambal River
c) Wular Lake

IASbaba
Web: http://ilp.iasbaba.com/ Score:
Email: ilp@iasbaba.com 0.00 / 150
Page 57
Exam Title : 2019 - Test 1-Polity & Curr...
Email : rahulsreedhar8787@gmail.com
Contact :

d) Kolleru Lake
Correct Answer: A
Explanation

Solution (a)

Saltwater crocodile

· It is the largest crocodile.

· It is also known as the estuarine crocodile, Indo-Pacific crocodile, marine crocodile, sea
crocodile

· It can live in marine environments, but usually resides in saline and brackish mangrove
swamps, estuaries, deltas, lagoons, and lower stretches of rivers.

· Bhitarkanika National Park is a very good place to sight the giant Salt Water Crocodile

· IUCN status: Least Concerned

THINK!

· Muggers

· Gharials

QUESTION 70. MTczODc4K1JhaHVsIFNyZWVkaGFyK3JhaHVsc3JlZWRoYXI4Nzg3QGdtYWlsLmNvbStRVUVTV


ElPTiA2OQ==
Recently, Formalin was in news, in the context of

a) Improvement of growth in broiler chickens


b) Preserving fishes to prevent spoilage and extend shelf life
c) Its hallucinogenic properties
d) Being a central nervous system (CNS) stimulant
Correct Answer: B
Explanation

Solution (b)

Formalin

· It is a form of formaldehyde, which is classified as a carcinogenic to humans.

· It contains 37 to 40 percent formaldehyde, and is used mostly in mortuaries and labs.

· Formalin has also been used to preserve fresh foods to prevent spoilage and extend shelf life,
even though its consumption is harmful to human beings.

· It is used in building materials and to produce many household products.

· It is used in pressed-wood products

IASbaba
Web: http://ilp.iasbaba.com/ Score:
Email: ilp@iasbaba.com 0.00 / 150
Page 58
Exam Title : 2019 - Test 1-Polity & Curr...
Email : rahulsreedhar8787@gmail.com
Contact :

Operation Sagar Rani

· To ensure safety and hygiene at fish handling and distribution centres

Source: https://www.thehindu.com/news/cities/chennai/the-stink-of-formalin-and-a-sea-
of-trouble/article24367738.ece

QUESTION 71. MTczODc4K1JhaHVsIFNyZWVkaGFyK3JhaHVsc3JlZWRoYXI4Nzg3QGdtYWlsLmNvbStRVUVTV


lPTiA3MA==
Consider the following statements about National Health Protection Scheme

1. It covers 10 crore poor and vulnerable families

2. It is a flagship programme under Ayushman Bharat Programme

3. It will provide coverage upto Rs 5 lakh per family per annum in secondary and tertiary care
institutions

Select the correct statements

a) 1 and 2
b) 2 and 3
c) 1 and 3
d) All of the above
Correct Answer: D

Explanation

Solution (d)

The Government announced two major initiatives in health sector, as part of Ayushman Bharat p
rogramme . It is aimed at making path breaking interventions to address health holistically, in
primary, secondary and tertiary care systems, covering both prevention and health promotion.

Health and Wellness Centre

· The National Health Policy, 2017 has envisioned Health and Wellness Centres as the
foundation of India’s health system.

· Under this 1.5 lakh centres will bring health care system closer to the homes of people.

· These centres will provide comprehensive health care, including for non-communicable
diseases and maternal and child health services.

· These centres will also provide free essential drugs and diagnostic services.

· The Budget has allocated Rs.1200 crore for this flagship programme .

· Contribution of private sector through CSR and philanthropic institutions in adopting these ce
ntres is also envisaged.

National Health Protection Scheme

IASbaba
Web: http://ilp.iasbaba.com/ Score:
Email: ilp@iasbaba.com 0.00 / 150
Page 59
Exam Title : 2019 - Test 1-Polity & Curr...
Email : rahulsreedhar8787@gmail.com
Contact :

· The second flagship programme under Ayushman Bharat is National Health Protection
Scheme, which will cover over 10 crore poor and vulnerable families (approximately 50 crore
beneficiaries) providing coverage upto 5 lakh rupees per family per year for secondary and
tertiary care hospitalization.

QUESTION 72. MTczODc4K1JhaHVsIFNyZWVkaGFyK3JhaHVsc3JlZWRoYXI4Nzg3QGdtYWlsLmNvbStRVUVTV


lPTiA3MQ==
‘38th Parallel North” formed the border between

a) North Sudan and South Sudan


b) North Korea and South Korea
c) Croatia and Bosnia & Herzegovina
d) Mainland Spain and Catalonia
Correct Answer: B

Explanation

Solution (b)

The 38th parallel north formed the border between North and South Korea prior to the Korean
War.

QUESTION 73. MTczODc4K1JhaHVsIFNyZWVkaGFyK3JhaHVsc3JlZWRoYXI4Nzg3QGdtYWlsLmNvbStRVUVTV


ElPTiA3Mg==
Consider the following statements about Montenegro

1. It opens to Adriatic Sea

2. It is bordered by Kosovo

3. It was part of Yugoslavia

Select the correct statements

a) 1 and 2
b) 2 and 3
c) 1 and 3
d) All of the above
Correct Answer: D
Explanation

Solution (d)

All the statements are correct.

IASbaba
Web: http://ilp.iasbaba.com/ Score:
Email: ilp@iasbaba.com 0.00 / 150
Page 60
Exam Title : 2019 - Test 1-Polity & Curr...
Email : rahulsreedhar8787@gmail.com
Contact :

Montenegro borders Croatia, Bosnia and Herzegovina, Serbia, Kosovo and Albania. It has a
coast on the Adriatic Sea to the southwest.

After World War I, Montenegro became part of Yugoslavia. Following the breakup of Yugoslavia,
the republics of Serbia and Montenegro together established a federation known as the Federal
Republic of Yugoslavia, which was renamed Serbia and Montenegro in 2003. On the basis of an
independence referendum held in May 2006, Montenegro declared independence on 3 June of
that year.

QUESTION 74. MTczODc4K1JhaHVsIFNyZWVkaGFyK3JhaHVsc3JlZWRoYXI4Nzg3QGdtYWlsLmNvbStRVUVTV


lPTiA3Mw==
‘Pradhan Mantri Ujjwala Yojana’ is associated with

a) Financial Inclusion
b) Digital Literacy
c) Rural Electrification
d) LPG Connections
Correct Answer: D
Explanation

Solution (d)

Pradhan Mantri Ujjwala Yojana

· It is a scheme of the Ministry of Petroleum & Natural Gas for providing LPG connections to
women from Below Poverty Line (BPL) households.

IASbaba
Web: http://ilp.iasbaba.com/ Score:
Email: ilp@iasbaba.com 0.00 / 150
Page 61
Exam Title : 2019 - Test 1-Polity & Curr...
Email : rahulsreedhar8787@gmail.com
Contact :

· Under the scheme, five crore LPG connections are to be provided to BPL households.

· It provides a financial support of Rs 1600 for each LPG connection to the BPL households.

· The administrative cost of Rs. 1600 per connection, which includes a cylinder, pressure
regulator, booklet, safety hose, etc. would be borne by the Government.

Identification

· The identification of eligible BPL families will be made in consultation with the State
Governments and the Union Territories.

· BPL is a person/ household who suffers from at least one deprivation under the Socio-
Economic Caste census (SECC) - 2011 Database.

THINK!

· 'Give it Up' scheme

· PAHAL ( Pratyaksh Hanstantrit Labh ) scheme

Source: https://www.thehindu.com/todays-paper/tp-opinion/a-clean-cooking-strategy/
article24394188.ece

QUESTION 75. MTczODc4K1JhaHVsIFNyZWVkaGFyK3JhaHVsc3JlZWRoYXI4Nzg3QGdtYWlsLmNvbStRVUVTV


ElPTiA3NA==
Recently, ‘Swat Valley’ was in news. Where is it located?

a) Jordan
b) Palestine
c) Syria
d) None of the above
Correct Answer: D
Explanation

Solution (d)

Swat Valley

· Swat is a valley and an administrative district in the Khyber Pakhtunkhwa province of


Pakistan.

· The region is inhabited largely by Pashtun people, except in the valley's uppermost
reaches, where the Kohistani people dominate.

IASbaba
Web: http://ilp.iasbaba.com/ Score:
Email: ilp@iasbaba.com 0.00 / 150
Page 62

You might also like